Sie sind auf Seite 1von 57

Which of the statements given

1. According to the Economic Survey above is/are correct?


2017-18, Indian economy has been (a) 1 only
facing (b) 2 only
(a) Simultaneous slump in (c) Both 1 and 2
savings and investment (d) Neither 1 nor 2
(b) Slump in savings but boom in
investment Answer: (c)
(c) Boom in savings but slump in Explanation:
investment Statement 1 is correct.
(d) Simultaneous boom in Citizens' stake in exercising
savings and investment accountability diminishes if they do not
pay in a visible and direct way for the
Answer: (a) services the State commits to providing.
Explanation: If a citizen does not pay, he becomes a
India did not experience slowdown during free rider (using the service without
the „lost decade‟, not during the East paying), and cannot complain if the State
Asian crisis, not even after India‟s own provides a poor-quality service. If he
balance-of-payments crisis in 1991. But exits (not using the service at all), he
there is a slowdown going on for last few loses interest in holding the State
years. The current slowdown – in which accountable. Only if he pays and uses
both investment and savings have the service will he try to hold the State
slumped – is the first in India‟s history. accountable. Hence the expression: no
Yet, because the investment decline has representation without taxation. In other
been so gradual, the magnitude of the words, taxation is the economic glue that
shortfall so far is relatively less severe. binds citizens to the State in a necessary
Source: Economic Survey 2017-18, two-way relationship.
Volume 1, Chapter 3, Page 48 Statement 2 is correct.
A progressive tax is a tax in which the
2. Taxation is not just a vehicle for average tax rate increases as the taxable
raising State revenues. It is also amount increases. Progressive taxes are
important for economic and political imposed in an attempt to reduce the tax
development. It is widely believed incidence of people with a lower ability to
that direct taxation is more pay, as such taxes shift the incidence
important than indirect taxation increasingly to those with a higher ability-
because to-pay. The opposite of a progressive tax
(1) Citizens' stake in exercising is a regressive tax, where the average
accountability diminishes if tax rate or burden decreases as an
they do not pay in a visible individual's ability to pay increases. A
and direct way for the direct tax is a tax imposed upon a person
services the State commits to or property as distinct from a tax imposed
providing. upon a transaction, which is described as
(2) Direct tax is progressive in an indirect tax.
nature and therefore helps in Source: Economic Survey 2018,
reducing inequality while the Volume 1, Chapter 4, Page 56
impact of indirect taxation is
felt more directly by the poor.

Prelim IAS Test Series (2019) – GS Test 15 (31.03.2019)


1
Economic Survey, Budget, India Year Book, Social Schemes
3. Devolution Reports of the Ministry Even though most States have
of Panchayati Raj (MoPR) give constituted SFCs, very few have
data about the devolution of accepted their recommendations in full or
taxation powers to the Local even to a significant extent, especially
Governments. Consider the those that carry financial implications for
following statements in this regard: them. As per the latest MoPR Devolution
(1) In many Indian States, Rural Report (2015-16) the percentage of
and Urban Local acceptance of such recommendations
Governments have not been varies from as low as 11 percent in
devolved enough taxation Karnataka to above 50 percent in West
powers. Bengal, Andhra Pradesh and Rajasthan
(2) The share of revenues to full acceptance in Kerala.
assigned to Local Source: Economic Survey 2018,
Governments in many Indian Volume 1, Chapter 4, Page 62
States are much less vis-à-
vis expenditure assignments. 4. The Economic Survey 2017-18
(3) Even though most Indian talks about four potential
States have constituted State challenges to India‟s development,
Finance Commissions one of which is hyper-globalisation.
(SFCs), very few seem to Which of the following statement is
have accepted their not correct in the context of hyper-
recommendations to a globalisation?
significant extent. (a) It means dramatic change in
Which of the statements given the size, scope and velocity
above are correct? of globalization.
(a) 1 and 2 only (b) This process began in the
(b) 1 and 3 only late 1950s and it continues
(c) 2 and 3 only into the beginning of the 21st
(d) 1, 2 and 3 century.
(c) It covers all three dimensions
Answer: (d) of economic globalization,
Explanation: cultural globalization, and
Statements 1 and 2 are correct. political globalization.
In many Indian States, rural and urban (d) This globalization has led to a
local governments have not been backlash in advanced
devolved enough taxation powers. countries, which may pose a
Successive Devolution Reports of the threat to India‟s growth story.
Ministry of Panchayati Raj (MoPR) show
that the share of revenues assigned to Answer: (b)
Local Governments in many Indian Explanation:
States are much less vis-à-vis Option (a) is correct.
expenditure assignments. From these Hyper Globalisation is characterised by a
reports, however, it seems that several more rapid trade integration. It is a type
States - notably Kerala, Maharashtra, of globalization aimed at the elimination
Karnataka, Gujarat and West Bengal - of all transaction costs associated with
are consistently improving on this front. the movement between the natural
Statement 3 is correct. borders of Nation States of goods,

Prelim IAS Test Series (2019) – GS Test 15 (31.03.2019)


2
Economic Survey, Budget, India Year Book, Social Schemes
services, capital and finance. In it, world (1) Average GDP growth in
trade has soared much more rapidly than agriculture has been
world GDP. It thus means dramatic declining over the decades.
change in the size, scope and velocity of (2) The volatility of agricultural
globalization. growth in India has declined
Option (b) is incorrect. substantially over time.
The era of hyper globalisation started Which of the statements given
around the early 1990‟s, when there was above is/are correct?
a rapid increase in world trade. (a) 1 only
Option (c) is correct. (b) 2 only
Three forces of hyper-globalization: (c) Both 1 and 2
(1) Economic force in which extensive (d) Neither 1 nor 2
growth in global trade creates cross-
border economic integration, Answer: (b)
(2) Human communications force via the Explanation:
Internet are changing norms of Statement 1 is incorrect.
human communication blurring social Real agricultural growth since 1960 has
barriers, and averaged about 2.8 percent in India. The
(3) Technological disruption force period before the Green Revolution saw
coming from new innovations in growth of less than 2 percent; the
technology driven by Internet-of- following period until 2004 yielded growth
Things (IoT), big data, and Artificial of 3 percent; in the period after the global
Intelligence (AI) bringing massive agricultural commodity surge, growth
economic and rapid social changes. increased to 3.6 percent.
The deep integration of hyper- Statement 2 is correct.
globalization conflicts with and The volatility of agricultural growth in
threatens the sovereignty of the India has declined substantially over
Nation State. time: from a standard deviation of 6.3
Option (d) is correct. percent between 1960 and 2004 to 2.9
Economic Survey says that this percent since 2004. In particular,
globalization has led to a backlash in production of cereals has become more
advanced countries reflected in the robust to drought. But levels of volatility
decline in world trade-GDP ratios since continue to be high and substantially
2011. This means that the trading higher than in China where the ups and
opportunities available to the countries downs have been virtually eliminated. An
that developed earlier, specifically the important contributing factor in India is
ability to export at double digit rates of that agriculture even today continues to
growth for three to four decades be vulnerable to the vagaries of weather
consistently, may no longer be available. because close to 52 percent (73.2 million
Source: Economic Survey 2018, hectares area of 141.4 million hectares
Volume 1, Chapter 5, Page 72 net sown area) of it is still un-irrigated
and rainfed.
5. India is still largely a country Source: Economic Survey 2018,
dependent on agriculture. Consider Volume 1, Chapter 6, Page 84.
the following statements regarding
agricultural economics of India: 6. With respect to the impact of
climate change on Indian

Prelim IAS Test Series (2019) – GS Test 15 (31.03.2019)


3
Economic Survey, Budget, India Year Book, Social Schemes
agriculture, consider the following against a backdrop of extreme
statements: groundwater depletion, especially in
(1) The impact of changes in North India.
temperature and rainfall is Source: Economic Survey 2018,
felt only in the extreme; that Volume 1, Chapter 6, Page 96-97
is, when temperatures are
much higher, rainfall is 7. The term „Son meta-preference‟ is
significantly lower. used in the Economic Survey
(2) Such impacts are 2017-18. Which of the following
significantly more adverse in statements is incorrect in this
unirrigated areas. context?
Which of the statements given (a) The term means that having
above is/are not correct? children until the desired
(a) 1 only number of sons are born.
(b) 2 only (b) It leads to sex selective
(c) Both 1 and 2 abortions leading to the
(d) Neither 1 nor 2 category of “missing” women.
(c) It leads to notional category
Answer: (d) of “unwanted” girls.
Explanation: (d) None of the above.
Economic Survey 2017-18 estimated the
impact of temperature and precipitation Answer: (b)
on agriculture. The main findings are as Explanation:
follows: Economic Survey 2017-18 says that the
• A key finding - and one with area where Indian society needs to
significant implications as climate reflect on the most is what might be
change looms - is that the impact of called “son preference” (not son-meta
temperature and rainfall is felt only in preference) where development is not
the extreme; that is, when proving to be an antidote. Son
temperatures are much higher, preference giving rise to sex selective
rainfall significantly lower, and the abortion and differential survival has led
number of “dry days” greater, than to skewed sex ratios at birth and beyond,
normal. Thus, statement 1 is leading to estimates of 63 million
correct. “missing” women. Hence, option (b) is
• A second key finding is that these incorrect.
impacts are significantly more There is another phenomenon of son
adverse in unirrigated areas (and meta-preference which involves parents
hence rain-fed crops such as pulses) adopting fertility “stopping rules” – having
compared to irrigated areas (and children until the desired number of sons
hence crops such as cereals). Thus are born. This meta-preference leads
statement 2 is also correct. naturally to the notional category of
The policy implications of these findings “unwanted” girls which is estimated at
are stark. India needs to spread irrigation over 21 million. In some sense, once
– and do so against a backdrop of rising born, the lives of women are improving
water scarcity and depleting groundwater but society still appears to want fewer of
resources. The challenge is that the them to be born.
spread of irrigation will have to occur

Prelim IAS Test Series (2019) – GS Test 15 (31.03.2019)


4
Economic Survey, Budget, India Year Book, Social Schemes
Source: Economic Survey 2018, fourths of all expenditures to about three-
Volume 1, Chapter 7, Page 105 fifths.
Statement 4 is incorrect.
8. Consider the following statements Government expenditure on R&D is
with respect to scientific research undertaken almost entirely by the Central
expenditure in India: Government. There is a need for greater
(1) Indian investment in State Government spending, especially
Research and Development application-oriented R&D aimed at
(R&D) is not increasing over problems specific to their economies and
the years, but has stagnated. populations.
(2) Indian investment in R&D as Source: Economic Survey 2018,
a percentage of GDP is Volume 1, Chapter 8, Page 120-121
decreasing over the years.
(3) Public sector investment in 9. Which of the following government
R&D is less than 50% of total programs provide avenues to
investment in R&D in India. qualified Indian researchers
(4) The Central government residing in foreign countries to work
spends as much on R&D in in Indian Institutes/Universities?
India as all the States (1) Ramanujan Fellowship
together in India. Scheme
Which of the statements given (2) INSPIRE Faculty Scheme
above are incorrect? (3) Ramalinga Swami Re-entry
(a) 1, 2 and 3 only Fellowship.
(b) 2, 3 and 4 only Select the correct answer using the
(c) 1, 3 and 4 only code given below:
(d) 1, 2, 3 and 4 (a) 1 and 2 only
(b) 2 and 3 only
Answer: (d) (c) 1 and 3 only
Explanation: (d) 1, 2 and 3
Statement 1 is incorrect.
Investments in Indian science, measured Answer: (d)
in terms of Gross Expenditure on R&D Explanation:
(GERD), have shown a consistently There are a number of government
increasing trend over the years. GERD programs such as the Ramanujan
has tripled in the last decade in nominal Fellowship Scheme, the Innovation in
terms – from Rs. 24,117 crores in 2004- Science Pursuit for Inspired Research
05 to Rs. 85,326 crores in 2014-15 and (INSPIRE) Faculty Scheme and the
an estimated Rs.1,04,864 crores in 2016- Ramalinga Swami Re-entry Fellowship,
17 – and double in real terms. that provide avenues to qualified Indian
Statement 2 is incorrect. researchers residing in foreign countries,
As a fraction of GDP, public expenditures to work in Indian institutes/universities.
on research have been stagnant – Statement 1 is correct.
between 0.6-0.7 percent of GDP – over Ramanujan Fellowship is meant for
the past two decades. brilliant scientists from all over the world
Statement 3 is incorrect. to take up scientific research positions in
Public expenditure is dominant, although India. The Fellows could work in any of
its share has come down from three-

Prelim IAS Test Series (2019) – GS Test 15 (31.03.2019)


5
Economic Survey, Budget, India Year Book, Social Schemes
the scientific institutions and universities Select the correct answer using the
in the country. code given below:
Statement 2 is correct. (a) 1 only
The INSPIRE Faculty Scheme, started in (b) 2 only
2011, has benefited over 1000 young (c) Both 1 and 2
scientists and over 450 have been (d) Neither 1 nor 2
absorbed as faculty in various
institutions. It is an opportunity for young Answer: (b)
achievers for developing independent Explanation:
scientific profiles and launch themselves Statement 1 is incorrect.
in fulfilling long-term careers. The Rural local self-governments collect less
scheme provides a research career than 10 percent of their total resources
opportunity for five years but no from own revenues (i.e. taxes) and
guarantee is provided for a job after Urban local self-governments around 45
completion of this period. percent.
Statement 3 is correct. Statement 2 is correct.
Ramalinga Swami Re-entry Faculty The property taxes collected at the
Fellowship of Department of second and third tiers of government are
Biotechnology (DBT) was meant to bring (a) land tax assessed and collected at
back Indian scientists working abroad so the state level; and
that they can pursue their research (b) building tax, including property/house
interests of national relevance. tax, collected at the municipality
Economic Survey suggests that these (ULG) and gram panchayat (RLG)
schemes could be enhanced to take levels.
advantage of opportunities to recruit in a Source: Economic Survey 2018,
way to build whole research groups; the Volume 1, Chapter 4, Page 62, 63
inducements should be such as to allow
them to do good research (laboratory 11. The Government of India is
resources, ability to hire post-docs, attempting the codification of the
housing etc.) rather than financial, to labour laws in order to remove the
ensure that home grown talent has a multiplicity of definitions and
level playing field. authorities and ease of compliance
Source: Economic Survey 2018, without compromising wage
Volume 1, Chapter 8, Page 129 security and social security. Which
of the following are the proposed
10. Which of the following statements codes?
regarding taxation by local (1) Code on Salaries
governments is/are correct? (2) Code on Safety and Working
(1) Tax collection of rural local Conditions
self-government as a ratio of (3) Code on Industrial Relations
their total resources is more (4) Code on Social and
than that of urban local self- Economic Security
government. Select the correct answer using the
(2) Property tax is both in the code given below:
domain of State government (a) 1 and 2 only
as well as the local self (b) 1 and 3 only
government. (c) 3 and 4 only

Prelim IAS Test Series (2019) – GS Test 15 (31.03.2019)


6
Economic Survey, Budget, India Year Book, Social Schemes
(d) 2 and 3 only Explanation:
Women in Politics 2017 Map is created
Answer: (d) by the Inter-Parliamentary Union (IPU)
Explanation: and UN Women. It depicts global
The Government is in the process of rankings for women in the executive and
rationalizing 38 Central Labour Acts by parliamentary branches of government
framing relevant provisions of existing as of 1 January 2017. The map shows
laws into 4 labour codes viz slow progress towards gender equality in
 Code on Wages these areas at regional and national
 Code on Safety and Working levels.
Conditions The Inter-Parliamentary Union (IPU) is
 Code on Industrial Relations a global inter-parliamentary institution
 Code on Social Security and Welfare. established in 1889 by Frédéric Passy
The draft Code on Wages Bill 2017 has (France) and William Randal Cremer
been introduced in Lok Sabha in August (United Kingdom). It was the first
2017 and referred to the Standing permanent forum for political multilateral
Committee on Labour for examination. negotiations. Initially, the organization
The other three Codes are at pre- was for individual parliamentarians, but
legislative consultation stage. has since transformed into an
Source: Economic Survey 2018, international organization of the
Volume 2, Chapter 10, Page 171 parliaments of sovereign states. The
national parliaments of 178 countries.
12. Women‟s political empowerment The IPU has permanent observer status
and equal access to leadership at the United Nations General Assembly.
positions at all levels are The United Nations Entity for Gender
fundamental to achieving the Equality and the Empowerment of
Sustainable Development Goals Women, also known as UN Women, is
(SDGs) and a more equal world. a United Nations entity working for the
With limited growth in women‟s empowerment of women. UN Women
representation, advancement of became operational in January 2011. UN
gender equality and the success of Women is a member of the United
the SDGs are jeopardized. Nations Development Group.
According to the Women in Politics Source: Economic Survey 2018,
2017 Map, the Lok Sabha had 64 Volume 2, Chapter 10, Page 173
(11.8 percent of 542 MPs) and the
Rajya Sabha had 27 (11 per cent 13. As per the United Nations National
of 245 MPs) women MPs. Who Accounts Statistics data based on
prepares the Women in Politics UN System of National Accounts,
Map? India‟s ranking improved from 14th
(a) Inter-Parliamentary Union position in 2006 to 7th position in
and UN Women. 2016, among the world‟s 15 largest
(b) Global Gender Empowerment economies in terms of overall GDP.
Association Consider the following statements
(c) UN Female Agenda regarding UN System of National
(d) World Political Forum Accounts:

Answer: (a)

Prelim IAS Test Series (2019) – GS Test 15 (31.03.2019)


7
Economic Survey, Budget, India Year Book, Social Schemes
(1) It is a mandatory system of Bank, the Organisation for Economic Co-
accounts for UN member operation and Development (OECD) and
countries. the Statistical Office of the European
(2) Along with the UN, it is also Communities.
maintained by the Source: Economic Survey 2018,
International Monetary Fund, Volume 2, Chapter 9, Page 153
the World Bank, the
Organisation for Economic 14. The Economic Survey 2017-18
Co-operation and quotes that as per the World
Development and the Investment Report, following a
Statistical Office of the surge in foreign investment in
European Communities. 2015, global FDI flows fell by 2 per
Which of the statements given cent. Who publishes the said World
above is/are not correct? Investment Report?
(a) 1 only (a) United Nations Conference
(b) 2 only on Trade and Development
(c) Both 1 and 2 (UNCTAD)
(d) Neither 1 nor 2 (b) World Economic Forum
(WEF)
Answer: (a) (c) World Bank (WB)
Explanation: (d) International Monetary Fund
The System of National Accounts (often (IMF)
abbreviated as SNA; formerly the United
Nations System of National Accounts or Answer: (a)
UNSNA) is an international standard Explanation:
system of national accounts, the first The United Nations Conference on Trade
international standard being published in and Development (UNCTAD) is the part
1953. The aim of SNA is to provide an of the United Nations Secretariat dealing
integrated, complete system of accounts with trade, investment, and development
enabling international comparisons of all issues. The organization's goals are to:
significant economic activity. "maximize the trade, investment and
Statement 1 is incorrect. development opportunities of developing
The suggestion is that individual countries and assist them in their efforts
countries use SNA as a guide in to integrate into the world economy on an
constructing their own national equitable basis". The UNCTAD was
accounting systems, to promote established by the United Nations
international comparability. However, General Assembly in 1964 and it reports
adherence to an international standard is to the UN General Assembly and United
entirely voluntary, and cannot be rigidly Nations Economic and Social Council.
enforced. The systems used by some The World Investment Report has been
countries (for example, France, United published annually since 1991 by the
States and China) differ significantly from UNCTAD. Each year´s Report covers the
the SNA. latest trends in foreign direct investment
Statement 2 is correct. around the World and analyses in depth
SNA continues to evolve and is one selected topic related to foreign
maintained by the United Nations, the direct investment and development.
International Monetary Fund, the World

Prelim IAS Test Series (2019) – GS Test 15 (31.03.2019)


8
Economic Survey, Budget, India Year Book, Social Schemes
Source: Economic Survey 2018, some people as a leading economic
Volume 2, Chapter 9, Page 153 indicator because it predicts future
economic activity.
15. The Baltic Dry Index, referred to in Source: Economic Survey 2018,
the Economic Survey 2017-18, is Volume 2, Chapter 9, Page 158
related to which of the following?
(a) It is an index used as a proxy 16. The Eight Core Industries comprise
for trade and shipping nearly 40.27% of the weight of
services. items included in the Index of
(b) It is an index used to study Industrial Production (IIP). These
variations in the sea surface include Electricity, steel, refinery
level in the Baltic sea. products, crude oil, coal, cement,
(c) It is an index used to natural gas and fertilisers. Which of
calculate precipitation in the the following correctly depicts them
Baltic sea on mean-year by decreasing order of weightage
basis. in IIP?
(d) It is a Humidity related index (a) Petroleum Refinery
in the northern hemisphere. Production, Electricity
Generation, Steel Production,
Answer: (a) Coal Production.
Explanation: (b) Electricity Generation,
The Baltic Dry Index (BDI) is a freight Petroleum Refinery
index and a good proxy for the Production, Steel Production,
robustness of trade and shipping Coal Production.
services. Most directly, the index (c) Steel Production, Petroleum
measures the demand for shipping Refinery Production,
capacity versus the supply of dry bulk Electricity Generation, Coal
carriers. The demand for shipping varies Production.
with the amount of cargo that is being (d) Coal Production, Petroleum
traded or moved in various markets Refinery Production,
(supply and demand). Electricity Generation, Steel
The Baltic Dry Index is issued daily by Production.
the London-based Baltic Exchange. The
BDI is a composite of the Capesize, Answer: (a)
Panamax and Supramax Timecharter Explanation:
Averages. It is reported around the world The Index of Industrial Production (IIP) is
as a proxy for dry bulk shipping stocks as an index for India which details out the
well as a general shipping market growth of various sectors in an economy
bellwether. such as mineral mining, electricity and
Because dry bulk primarily consists of manufacturing. The all India IIP is a
materials that function as raw material composite indicator that measures the
inputs to the production of intermediate short-term changes in the volume of
or finished goods, such as concrete, production of a basket of industrial
electricity, steel, and food; the index is products during a given period with
also seen as an efficient economic respect to that in a chosen base period. It
indicator of future economic growth and is compiled and published monthly by the
production. The BDI is considered by Central Statistical Organisation (CSO),

Prelim IAS Test Series (2019) – GS Test 15 (31.03.2019)


9
Economic Survey, Budget, India Year Book, Social Schemes
Ministry of Statistics and Programme covers specified goods originating in or
Implementation six weeks after the exported from named countries or
reference month ends. exporters. ADD is chargeable in addition
The Eight Core Industries comprise to, and independent of, any other duty to
nearly 40.27% of the weight of items which the imported goods are liable.
included in the Index of Industrial Statement 2 is incorrect.
Production (IIP). These are Electricity, Countervailing duty is also a customs
steel, refinery products, crude oil, coal, duty on goods that have received
cement, natural gas and fertilisers. government subsidies in the originating
Petroleum Refinery production (28.04%), or exporting country. For customs
Electricity generation (19.85%), Steel purposes, it is treated in the same way
production (17.92%), Coal production as ADD. It is possible to have both ADD
(10.33%), Crude Oil production (8.98%), and Countervailing Duty on a product.
Natural Gas production (6.88%), Cement Source: Economic Survey 2018,
production (5.37%), Fertilizers production Volume 2, Chapter 8, Page 126
(2.63%)
Source: Economic Survey 2018, 18. Which of the following are the
Volume 2, Chapter 8, Page 121-122 attempts by the Government of
India to mobilise savings in the
17. The Economic Survey 2018 economy?
mentions that the Government (1) Initiatives to unearth black
notified anti-dumping duties and money.
countervailing duties on various (2) Encouraging the conversion
steel products. Which of the of gold into financial
following statements is/are correct instruments.
in this context? (3) Initiatives to increase Foreign
(1) Anti-dumping duty is a type Direct Investments (FDIs).
of customs duty. (4) Popularizing Sukanya
(2) Countervailing duty is a type Samriddhi Scheme under
of excise duty. Beti Bachao Beti Padhao
Select the correct answer using the campaign.
code given below: Select the correct answer using the
(a) 1 only code given below:
(b) 2 only (a) 1, 2 and 3 only
(c) Both 1 and 2 (b) 2, 3 and 4 only
(d) Neither 1 nor 2 (c) 1, 3 and 4 only
(d) 1, 2, 3 and 4
Answer: (a)
Explanation: Answer: (d)
Statement 1 is correct. Explanation:
An anti-dumping duty (ADD) is a customs Statement 1 is correct.
duty on imports providing a protection Initiatives to unearth black money (e.g.
against the dumping of goods at prices demonetisation) – it brings the money
substantially lower than the normal value. into „white economy‟ where it is
In most cases, this is the price foreign accounted for, taxed and mostly put into
producers charge for comparable sales the bank accounts. Thus, its channelized
in the producer‟s own country. Each ADD into savings for the nation.

Prelim IAS Test Series (2019) – GS Test 15 (31.03.2019)


10
Economic Survey, Budget, India Year Book, Social Schemes
Statement 2 is also correct. (a) 1 and 2 only
Conversion of gold into financial (b) 1, 2 and 3 only
instruments like gold bond or paper (c) 2, 3 and 4 only
brings investment into physical movable (d) 1, 2, 3 and 4
property into financial savings.
Statement 3 is correct. Answer: (d)
Seeking FDI into India is nothing but Explanation:
channelizing savings in some other All the given statements are correct.
country into our country. Thus, it is also Since 1980, India has been rapidly
an attempt to channelize foreign savings. catching up, posting an average per
Statement 4 is correct. capita GDP growth rate of 4.5 percent, a
Sukanya Samriddhi Yojana is a small rate substantially greater than registered
savings scheme that gives competitive previously, which is in the top quartile of
interest and options to people in long run, countries over that period, and amongst
thus attracting household money into the highest for continuous democracies.
national savings. But this fast growth has occurred with
Source: Economic Survey 2018, limited transfer of labour resources from
Volume 1, Chapter 3, Page 53 low productivity to high productivity and
dynamic sectors, and despite relatively
19. India has been one of the most modest agricultural growth. The risk for
dynamic economic performers in India–as for the other late convergers–is
the world. A major driver of these that resources (especially labour) will
good times, is “economic move from low productivity, informal
convergence,” whereby poorer sectors to other sectors that are
countries like India have grown marginally less formal and only
faster than richer countries and marginally more productive. That is the
closed the gap in standards of “late convergerstall” that India must
living. Which of the following could avoid. Rapidly improving human capital–
be potential factors creating barrier – healthy individuals, including all
to further growth and for slowdown women, with the basic education to
in this process of economic continually learn and adapt––will be key
convergence? to sustaining India‟s dynamic growth
(1) Backlash against trajectory. Rapidly improving agricultural
globalization which reduces productivity––against the headwinds of
exporting opportunities. climate change and water scarcity––will
(2) Difficulties of transferring be another key to achieving good growth
resources from low and hence sustainable growth. And the
productivity to higher hyperglobalization backlash in advanced
productivity sectors. countries, over which India has little
(3) Challenge of upgrading control, must recede to create a
human capital to the favourable external climate to sustain
demands of a technology- rapid growth. There is no Late Converger
intensive workplace. Stall, as yet, but it would be wise to act to
(4) Climate change-induced head it off.
agricultural stress. Source: Economic Survey 2018,
Select the correct answer using the Volume 1, Chapter 5, Page 80
code given below:

Prelim IAS Test Series (2019) – GS Test 15 (31.03.2019)


11
Economic Survey, Budget, India Year Book, Social Schemes
20. Recently the Prime Minister of Sources:
India served the three billionth https://www.akshayapatra.org/indias-
meal of Akshaya Patra to school mid-day-meal-scheme
children. In this context consider https://mhrd.gov.in/mid-day-meal
the following statements:
(1) Akshaya Patra Foundation is 21. In the context of „Interim Budget‟
a semi-government consider the following statements:
organization. (1) In Interim Budget, the
(2) Mid-Day Meal is a scheme of Government presents
the Ministry of Women & estimates only for a part of
Child Development. the year.
(3) The Mid-Day Meal Scheme is (2) In Interim Budget, the
applicable to recognized as Government is prohibited
well as unrecognized from making mega-
Madarsas/Maqtabs. announcements.
(4) The Mid-Day Meal Scheme is Which of the statement given
covered by the National Food above is/are incorrect?
Security Act, 2013 (a) 1 only
Which of the statements given (b) 2 only
above is/are correct? (c) Both 1 and 2
(a) 1 and 2 only (d) Neither 1 nor 2
(b) 2 and 3 only
(c) 3 and 4 only Answer: (c)
(d) 1, 2, 3 and 4 Explanation:
Statement 1 is incorrect.
Answer: (c) In an interim Budget, the vote-on-account
Explanation: seeks the Parliament‟s nod for incurring
Statements 1 and 2 are incorrect and expenditure for part of a fiscal year.
statements 3 and 4 are correct. However, the estimates are presented for
Akshaya Patra Foundation is a the entire year, as is the case with the
Bengaluru-based not-for-profit regular Budget. However, the incoming
organization Funded by International government has full freedom to change
Society for Krishna Consciousness the estimates completely when the final
(ISKCON). Hence, statement 1 is Budget is presented.
incorrect. Statement 2 is incorrect.
The Mid-Day Meal is a scheme of Further, there are no prohibitions on the
Ministry of Human Resources government from making any mega-
Development (MHRD). Hence, announcements. It is only by convention
statement 2 is incorrect. that Governments refrain from making
Since April 2008 the scheme is big-budget announcements.
applicable to recognized as well as
unrecognized Madarsas/Maqtabs 22. Which of the following statements
supported under Sarva Shiksha Abhiyan is/are incorrect regarding the Atal
(SSA). Hence, statement 3 is correct. Pension Yojana (APY)?
The Mid-Day Meal Scheme is covered by (a) The age of exiting the
the National Food Security Act, 2013. scheme and the start of the
Hence, statement 4 is correct. pension is 60 years.

Prelim IAS Test Series (2019) – GS Test 15 (31.03.2019)


12
Economic Survey, Budget, India Year Book, Social Schemes
(b) Minimum age of joining the (2) Presentation of macro-
Atal Pension Yojana is 18 economic policy framework
years and maximum age is and fiscal policy strategy
50 years. statement in the Annual
(c) The Central government Budget are mandated by the
would co-contribute 50% of Fiscal Responsibility and
the total contribution or Rs. Budget Management (FRBM)
1000 per annum whichever is Act.
lower to the subscriber‟s Which of the statements given
account. above is/are correct?
(d) Small Finance Banks are (a) 1 only
allowed to offer APY. (b) 2 only
(c) Both 1 and 2
Answer: (b) (d) Neither 1 nor 2
Explanation:
The Atal Pension Yojana (previously Answer: (c)
known as Swavalamban Yojana) is a Explanation:
government-backed pension scheme Presentation of the Annual Financial
targeted at the unorganised sector. It Statement and Demand for Grants in the
was launched by the Prime Minister Annual Budget are mandated by the
Narendra Modi in May 2015. As of May Constitution of India under Article 112
2015, only 20% of India's population had and 113. Presentation of macro-
any kind of pension scheme, this scheme economic policy framework and fiscal
aims to increase the number. policy strategy statement along with
 Minimum age of joining the Atal medium term fiscal policy statement in
Pension Yojana is 18 years and the Annual Budget are mandated by the
maximum age is 40 years. Fiscal Responsibility and Budget
 The age of exiting the scheme and Management (FRBM) Act, 2003.
the start of pension is 60 years.
 The Central government would co- 24. Which of the following statements
contribute 50% of the total regarding the VAJRA initiative is
contribution or Rs. 1000 per annum correct?
whichever is lower to the subscriber‟s (a) It is an “advanced shield of
account. Indian army against missile
 Small Finance Banks are allowed to attack”
offer APY. (b) It is a programme of CDAC to
protect Indian cyber
23. In the Annual Budget various infrastructure against cyber-
documents are presented by the attack.
Government of India. In this (c) It is an initiative to connect
context consider the following the Indian academic and
statements: Research and Development
(1) Presentation of the 'Annual (R&D) systems to the best of
Financial Statement' and global science and scientists
'Demand for Grants' in the for a sustained international
Annual Budget are mandated collaborative research.
by the Constitution of India.

Prelim IAS Test Series (2019) – GS Test 15 (31.03.2019)


13
Economic Survey, Budget, India Year Book, Social Schemes
(d) It is a project launched by the of the subsequent month. Presently, call
Government of India in for application is made open for
Antarctica for R&D on climate prospective researchers.
change and its impact on Source:
oceans. http://pib.nic.in/newsite/PrintRelease.asp
x?relid=169081
Answer: (c)
Explanation: 25. Consider the following statements
The Science and Engineering Research regarding the Negotiable
Board (SERB), a Statutory body of the Warehouse Receipts (NWRs):
Department of Science and Technology (1) NWRs can be traded online.
(DST) has recently launched a „Visiting (2) NWRs issued by registered
Advanced Joint Research (VAJRA) warehouses help farmers to
Faculty Scheme‟ to connect the Indian seek loans from banks
academic and research and development against NWRs.
(R&D) systems to the best of global (3) NWRs allow transfer of
science and scientists for a sustained ownership of that commodity
international collaborative research. stored in a warehouse
The scheme offers adjunct/visiting faculty without having to deliver the
assignments to overseas scientists, physical commodity.
faculty members and R&D professionals Which of the statements given
including Non-Resident Indians (NRI) above are correct?
and Overseas Citizen of India (OCI) to (a) 1 and 2 only
undertake high quality collaborative (b) 1 and 3 only
research in cutting edge areas of science (c) 2 and 3 only
and technology including interdisciplinary (d) 1, 2 and 3
areas of priority such as energy, water,
health, security, nutrition, materials and Answer: (d)
manufacturing, etc. with one or more Explanation:
Indian collaborators of public funded In India, the term „Negotiable Warehouse
academic and research institutions of Receipt‟ (NWR) is defined in Section
India. 2(m) of the Warehousing (Development
The VAJRA Faculty should be an active and Regulation) Act, 2007 (WDR Act),
researcher working in an overseas which came into force from 25 October
leading academic/research/industrial 2010. The WDR Act provides for
organization with significant issuance of Negotiable Warehouse
accomplishments in R&D. The initial Receipts (NWRs) by the warehouses
faculty assignment is given for a period registered under this Act.
of one year extendable to subsequent Statement 1, 2 and 3 are correct.
years based on the collaborative These NWRs can be traded online.
outcome and interest. The residency Further, the NWRs issued by registered
period of VAJRA Faculty in the host warehouses help farmers to seek loans
institution would be for a minimum of 1 from banks against NWRs. NWRs allow
month and a maximum of 3 months transfer of ownership of that commodity
every year. VAJRA Faculty is provided stored in a warehouse without having to
US Dollars 15000 in the first month of deliver the physical commodity. These
residence and US Dollars 10000 in each receipts are issued in negotiable form,

Prelim IAS Test Series (2019) – GS Test 15 (31.03.2019)


14
Economic Survey, Budget, India Year Book, Social Schemes
making them eligible as collateral for Statement 1 is incorrect.
loans The scheme has the benefit cover of Rs.
Source: 5 lakhs per family per year. The target
https://www.thehindubusinessline.co beneficiaries of the proposed scheme will
m/economy/agri-business/govt- be more than 10 crore families belonging
launches-negotiable-warehouse- to poor and vulnerable population based
receipts/article23049051.ece on SECC database. AB-NHPM will
subsume the on-going centrally
26. Consider the following statements sponsored schemes - Rashtriya
regarding the Ayushman Bharat- Swasthya Bima Yojana (RSBY) and the
National Health Protection Mission Senior Citizen Health Insurance Scheme
(AB-NHPM): (SCHIS).
(1) The scheme provides benefit Statement 2 is correct.
cover of Rs. 5 lakhs per AB-NHPM will have a defined benefit
person per year belonging to cover of Rs. 5 lakhs per family per year.
the poor and vulnerable This cover will take care of almost all
population based on the secondary care and most of tertiary care
Socio Economic and Caste procedures. To ensure that nobody is left
Census (SECC) 2011 out (especially women, children and
database. elderly) there will be no cap on family
(2) There will be no cap on size and age in the scheme. The benefit
family size and age as per cover will also include pre and post-
the scheme. hospitalisation expenses. All pre-existing
(3) Benefits of the scheme are conditions will be covered from day one
portable across the country of the policy. A defined transport
and a beneficiary covered allowance per hospitalization will also be
under the scheme will be paid to the beneficiary.
allowed to take cashless Statement 3 is correct.
benefits from any Benefits of the scheme are portable
public/private empanelled across the country and a beneficiary
hospitals across the country. covered under the scheme will be
Which of the statements given allowed to take cashless benefits from
above is/are correct? any public/private empanelled hospitals
(a) 1 only across the country.
(b) 2 and 3 only Source:
(c) 1 and 3 only http://pib.nic.in/newsite/PrintRelease.a
(d) 3 only spx?relid=177817

Answer: (b) 27. Engel‟s law is seen in the news


Explanation: sometimes. It is related to which of
The Union Cabinet launched a new the following?
Centrally Sponsored scheme: Ayushman (a) Consumer expenditure
Bharat - National Health Protection behaviour
Mission (AB-NHPM) having central (b) Unemployment pattern in the
sector component under Ayushman economy
Bharat Mission anchored in the Ministry (c) Import-export profile of the
of Health and Family Welfare (MoHFW). economy

Prelim IAS Test Series (2019) – GS Test 15 (31.03.2019)


15
Economic Survey, Budget, India Year Book, Social Schemes
(d) Liquidity in the Banking agricultural systems to effectively support
Sector development and ensure food security in
a changing climate.
Solution: (a) Objectives of CSA are:
Explanation:  To sustainably increase agricultural
Engel's law is an observation in productivity and incomes.
economics stating that as income rises,  Adapting and building resilience to
the proportion of income spent on food climate change.
falls, even if absolute expenditure on  Reducing and/or removing
food rises. In other words, the income greenhouse gas emissions wherever
elasticity of demand of food is between 0 possible.
and 1. Engel's law does not imply that Source: Economic Survey 2017-18,
food spending remains unchanged as Volume II, Chapter 7: Agriculture and
income increases: It suggests that Food Management, Page No. 113
consumers increase their expenditures
for food products in percentage terms 29. Which among the following is/are
less than their increases in income. the benefits of adopting safe
Source: Economic Survey 2017-18, harbour rules?
Volume II, Chapter 1: An Overview of (1) Advance information or
India‟s Economic Performance in knowledge about the range
2017-18, Page No. 8 of profits or prices to qualify
https://www.investopedia.com/terms/e for the safe.
/engels-law.asp (2) Elimination of the possibility
of litigation between
28. Which of the following is/are the taxpayers and revenue
objectives of Climate-Smart authorities.
Agriculture? (3) Automatic approvals and
(1) Sustainably increase self-assessment procedures.
agricultural productivity and Select the correct answer using the
incomes. code given below:
(2) Adapting and building (a) 1 only
resilience to climate change. (b) 1 and 2 only
(3) Reducing and/or removing (c) 2 and 3 only
greenhouse gas emissions (d) 1, 2 and 3
wherever possible.
Select the correct answer using the Answer: (d)
code given below: Explanation:
(a) 1 only A safe harbour is a provision of a statute
(b) 1 and 2 only or a regulation that specifies that certain
(c) 2 and 3 only conduct will be deemed not to violate a
(d) 1, 2 and 3 given rule.
From the perspective of transfer pricing
Answer: (d) provisions, the safe harbour rules provide
Explanation: a window for the taxpayers wherein in
Climate-smart agriculture (CSA) is an case of defined circumstances the
approach that helps to guide actions income-tax authorities shall accept the
needed to transform and reorient transfer pricing declared by the taxpayer.

Prelim IAS Test Series (2019) – GS Test 15 (31.03.2019)


16
Economic Survey, Budget, India Year Book, Social Schemes
The adoption of safe harbour rules enterprises in
provides many perceived benefits both the non-farm
for taxpayers and the revenue authorities sector
like- Which of the pairs given above
 Advance information or knowledge is/are correctly matched?
about the range of profits or prices to (a) 1 and 2 only
qualify for the safe. This brings (b) 2 and 3 only
certainty in transactions. (c) 3 only
 Elimination of the possibility of (d) 1, 2 and 3
litigation between taxpayers and
revenue authorities. Answer: (a)
 Automatic approvals and self- Explanation:
assessment procedures. Various schemes for the development
Source: Economic Survey 2017-18 of MSME sector-
Volume II: Chapter 02: Review of 1. Prime Minister‟s Employment
Fiscal Developments, Page No. 39. Generation Programme (PMEGP) is
https://www.bizsolindia.com/an- aimed at generating self-employment
overview-of-safe-harbour-rules-in- opportunities through establishment
indian-transfer-pricing-regime-april- of micro-enterprises in the non-farm
2014/ sector.
2. Credit Guarantee Scheme for MSME
30. Identify from the following, the – It covers collateral free credit facility
correct pairs of schemes for extended by eligible financial
MSMEs and the purpose of that institutions.
scheme: 3. Credit Linked Capital Subsidy
Scheme Details Scheme (CLCSS) aims at facilitating
1. Credit : It covers technology upgradation of the MSME
Guarantee collateral free sector.
Scheme credit facility 4. Pradhan Mantri Mudra Yojana: To
for MSME extended by provide funding to the non-corporate
eligible small business sector.
financial Source: Economic Survey 2017-18
institutions. Volume 2, Chapter 08, Industry and
2. Credit : It aims at Infrastructure, Page No. 128
Linked facilitating
Capital technology 31. Consider the following statements
Subsidy upgradation regarding growth indicators in
Scheme of the MSME India:
(CLCSS) sector. (1) Within manufacturing, growth
3. Pradhan : Aimed at is highest in sectors that are
Mantri generating relatively labour intensive.
Mudra self- (2) India‟s tax-GDP ratio of
Yojana employment around 17 per cent is half the
opportunities average of OECD countries.
through Which of the statements given
establishment above is/are correct?
of micro- (a) 1 only

Prelim IAS Test Series (2019) – GS Test 15 (31.03.2019)


17
Economic Survey, Budget, India Year Book, Social Schemes
(b) 2 only (d) 1, 2, 3 and 4
(c) Both 1 and 2
(d) Neither 1 nor 2 Answer: (d)
Explanation:
Answer: (b) Point 1 is correct:
Explanation: Tax-GDP ratio: To enhance public
Statement 1 is incorrect: investment, India should aim to increase
The share of manufacturing in India‟s its tax-GDP ratio to at least 22 per cent of
GDP is low relative to the average in low GDP by 2022- 23. Demonetization and
and middle-income countries. Within GST will contribute positively to this
manufacturing, growth has often been critical effort. In addition, efforts need to
highest in sectors that are relatively be made to rationalize direct taxes for
capital intensive, such as automobiles both corporate tax and personal income
and pharmaceuticals. This stems from tax.
India‟s inability to capitalize fully on its Point 2 is correct:
inherent labour and skill cost advantages FDI norms: Domestic savings can be
to develop large-scale labour-intensive complemented by attracting foreign
manufacturing. investment in bonds and government
Statement 2 is correct: securities. Regulatory limits can be
India‟s tax-GDP ratio of around 17 per relaxed for rupee denominated debt.
cent is half the average of OECD Point 3 is correct:
countries (35 per cent) and is low even Exit CPSEs: Inefficient CPSEs surviving
when compared to other emerging on government support distort entire
economies like Brazil (34 per cent), sectors as they operate without any real
South Africa (27 per cent) and China (22 budget constraints. The government‟s
per cent). To enhance public investment, exit will attract private investment and
India should aim to increase its tax-GDP contribute to the exchequer, enabling
ratio to at least 22 per cent of GDP by higher public investment.
2022- 23. Point 4 is correct:
Source: Strategy for New India @ 75 Debt-GDP ratio: lowering of the
by NITI Aayog; chapter 1: growth government debt-to-GDP ratio will help
reduce the relatively high interest cost
32. For increasing the investment rate, burden on the government budget, bring
NITI Aayog has suggested to: the size of India‟s government debt
(1) Increase the tax-GDP ratio. closer to that of other emerging market
(2) Liberalize FDI norms across economies, and improve the availability
sectors. of credit for the private sector in the
(3) Continue to exit Central financial markets.
Public Sector Enterprises Source: Strategy for New India @ 75
(CPSEs) that are not by NITI Aayog; Chapter 1: growth
strategic in nature.
(4) Lower the debt-GDP ratio. 33. The way forward for employment
Which of the statements given and labour reforms as given by the
above are correct? NITI Aayog is given below:
(a) 1 and 2 only (1) Increase female labour force
(b) 1, 2 and 3 only participation to at least 30
(c) 2, 3 and 4 only per cent by 2022-23.

Prelim IAS Test Series (2019) – GS Test 15 (31.03.2019)


18
Economic Survey, Budget, India Year Book, Social Schemes
(2) Expand the Minimum Wages well as recreation and sanitation
Act, 1948, to cover all jobs. facilities.
(3) Enforce the payment of  Increase female labour force
wages through cheque or participation to at least 30 per cent by
Aadhaar-enabled payments 2022-23.
for all. Source: Strategy for New India @ 75
(4) Make compliance with the by NITI Aayog; Chapter 2:
national floor level minimum employment and labour reforms
wage mandatory.
Which of the statements given 34. Global Innovation Index is
above is/are incorrect? published by
(a) 2 and 3 only (a) World Bank
(b) 3 only (b) UNDP (United Nations
(c) 1 and 4 only Development Programme)
(d) None of the above (c) WIPO (World Intellectual
Property Organisation)
Answer: (d) (d) WEF (World Economic
Explanation: Forum)
The suggestions given by NITI are as
follows: Answer: (c)
 Labour Market Information System Explanation:
(LMIS) should be made functional Global Innovation Index is published by
urgently. the World Intellectual Property
 Ensure the wider use of Organization jointly with Cornell
apprenticeship programmes by all University and INSEAD.
enterprises. WIPO: The World Intellectual Property
 Complete the codification of labour Organization is one of the 15 specialized
laws at the earliest. agencies of the United Nations. WIPO
 Ensure that data collection for the was created in 1967 "to encourage
Periodic Labour Force Survey (PFLS) creative activity, to promote the
of households initiated in April 2017 protection of intellectual property
is completed as per schedule and throughout the world".
data disseminated by 2019. Objective: India should be among the
 Ease industrial relations to top 50 countries in the Global Innovation
encourage Formalization Index by 2022-23.
 Make compliance with the national Source: Strategy for New India @ 75
floor level minimum wage mandatory. by NITI Aayog; Chapter 3: technology
 Expand the Minimum Wages Act, and innovation
1948, to cover all jobs.
 Enforce the payment of wages 35. Consider the following statements
through cheque or Aadhaar-enabled with regard to India‟s technology
payments for all. and innovation sector:
(1) R&D (Research and
 Ensure compulsory registration of all
Development) expenditure in
establishments to ensure better
India as a percentage of
monitoring of occupational safety as
GDP is less than 1 percent.

Prelim IAS Test Series (2019) – GS Test 15 (31.03.2019)


19
Economic Survey, Budget, India Year Book, Social Schemes
(2) The share of the private (c) It‟s a nutrition model
sector in R&D (Research and successfully run in
Development) investment is malnutrition areas.
more than 50% in India. (d) None of the above.
Which of the statements given
above is/are correct? Answer: (b)
(a) 1 only Explanation:
(b) 2 only The Madhepura Electric Locomotive
(c) Both 1 and 2 Project, a joint venture between the
(d) Neither 1 nor 2 Indian Railways and the French
multinational Alstom, provides a good
Answer: (a) example of how mega projects can be
Explanation: leveraged to boost domestic production.
Statement 1 is correct: The project enabled effective transfer of
The latest R&D statistics released by the technology and the availability of state-
National Science and Technology of-the-art locomotives for the railways.
Management Information System The Madhepura model is replicable in the
(NSTMIS) of the Department of Science defence, aerospace, railways and
and Technology (DST) show that while shipping sectors.
R&D expenditure in India tripled in the Source: Strategy for New India @ 75
period from 2004-05 to 2014-15, its size by NITI Aayog; Chapter 4: industry
as a percentage of GDP remained at 0.7
per cent. This is very low compared to 37. Identify the correctly matched
the 2 per cent and 1.2 per cent spent by pairs:
China (for 2015) and Brazil (for 2014) List I : List II
respectively. Countries like Israel spend 1. SAMPADA : Agro-marine
as much as 4.3 per cent of their GDP on processing
R&D. 2. PARIVESH : Logistics
Statement 2 is incorrect: Furthermore, 3. POSHAN : Nutrition
while the share of the private sector in 4. SAUBHAGYA : Electrificatio
R&D investment in most technologically n
advanced countries is as high as 65 per Select the correct answer using the
cent to 75 per cent, it is only about 30 per code given below:
cent in India. (a) 1 only
Source: Strategy for New India @ 75 (b) 2 only
by NITI Aayog; Chapter 3: technology (c) 1, 2 and 4 only
and innovation (d) 1, 3 and 4 only

36. What is the Madhepura model in Answer: (d)


the context of Indian economic Explanation:
sector? 1. Sampada scheme: SAMPADA
(a) It‟s a banking model of (Scheme for Agro-Marine Processing
Regional Rural Banks. and Development of Agro-Processing
(b) It‟s a model for locomotive Clusters) is an umbrella scheme
production. incorporating ongoing schemes of the
Ministry like Mega Food Parks,
Integrated Cold Chain and Value

Prelim IAS Test Series (2019) – GS Test 15 (31.03.2019)


20
Economic Survey, Budget, India Year Book, Social Schemes
Addition Infrastructure, Food Safety (d) The buyer cannot raise a
and Quality Assurance Infrastructure, permanent structure on the
etc. and also new schemes like producer‟s land.
Infrastructure for Agro-processing
Clusters, Creation of Backward and Answer: (a)
Forward Linkages, Creation / Explanation:
Expansion of Food Processing & Option (c) is correct - The draft Model
Preservation Capacities. Act was released by the Ministry of
2. Parivesh: PARIVESH (Pro-Active Agriculture and seeks to create a
and Responsive facilitation by regulatory and policy framework for
Interactive, Virtuous and contract farming. Based on this draft
Environmental Single-window Hub) Model Act, legislatures of States can
has been launched on August 10, enact a law on contract farming as
2018 as a single window hub for contracts fall under the Concurrent List of
environment, forest, wildlife and the Constitution. In this context, we
CRZ(Coastal Regulation Zone) discuss contract farming, issues related
clearances. to it, and progress so far.
3. Poshan Abhiyaan: Poshan (Prime What is contract farming?
Minister‟s Overarching Scheme for Under contract farming, agricultural
Holistic Nutrition) aims to ensure production (including livestock and
holistic development and adequate poultry) can be carried out based on a
nutrition for pregnant women, pre-harvest agreement between buyers
mothers and children. (such as food processing units and
4. Saubhagya scheme: exporters), and producers (farmers or
The Saubhagya Scheme or Pradhan farmer organisations). The producer can
Mantri Sahaj Bijli Har Ghar Yojana is sell the agricultural produce at a specific
an Indian government project to price in the future to the buyer as per the
provide electricity to all households. agreement. Under contract farming, the
Source: Strategy for New India @ 75 producer can reduce the risk of
by NITI Aayog; fluctuating market price and demand.
The buyer can reduce the risk of non-
38. Regarding the draft Model Contract availability of quality produce.
Farming Act, 2018, which of the Under the draft Model Act, the producer
following is incorrect? can get support from the buyer for
(a) Rights or title ownership of improving production through inputs
the producer‟s land can be (such as technology, pre-harvest and
transferred to the buyer. post-harvest infrastructure) as per the
(b) The Act takes contract agreement.
farming out of the ambit of Option (b) is correct - The Act takes
Agricultural Produce contract farming out of the ambit of
Marketing Committees Agricultural Produce Marketing
(APMCs). Committees (APMCs).
(c) The Ministry of Agriculture Option (d) is correct - However, the
has released the draft Model buyer cannot raise a permanent structure
Contract Farming Act, 2018. on the producer‟s land.

Prelim IAS Test Series (2019) – GS Test 15 (31.03.2019)


21
Economic Survey, Budget, India Year Book, Social Schemes
Option (a) is incorrect – Also rights or Doing Business Report is published by
title ownership of the producer‟s land the World Bank.
cannot be transferred to the buyer. Source: Strategy for New India @ 75
Source: Strategy for New India @ 75 by NITI Aayog (Multiple Chapters)
by NITI Aayog; Chapter 6: doubling
farmers‟ incomes 40. In the context of mineral resources
in India, consider the following
39. Consider the following pairs: statements:
List I : List II (1) Only 10 per cent of Obvious
1. Global : World Bank Geological Potential (OGP)
Findex area has been explored in
Database India and 1.5 per cent is
2017 being mined.
2. Composit : IPCC (2) Major minerals production is
e Water (Intergovernm more labour intensive as
Manage ental Panel on compared to minor minerals.
ment Climate Which of the statements given
Index Change) above is/are correct?
3. Global : World (a) 1 only
Competiti Economic (b) 2 only
veness Forum (c) Both 1 and 2
Index (d) Neither 1 nor 2
4. Doing : UNCTAD
Business (United Answer: (a)
Report Nations Explanation:
Conference Current status of mineral resource
on Trade and sector in India:
Development)  India has identified 5.71 lakh sq. km
Which of the pairs given above are as the obvious geological potential
correctly matched? (OGP) area, but only 10 per cent of it
(a) 1 and 3 only has been explored and 1.5 per cent
(b) 2 and 3 only is being mined. Hence, statement 1
(c) 1, 2 and 3 only is correct.
(d) 1, 2, 3 and 4  India also has large potential to
increase the production of minor
Answer: (a) minerals. It is estimated that their
Explanation: share in the value of production is
Pair 1 is correctly matched: about 26 per cent. Even though
Global Findex Database 2017 was minor minerals have a small share in
published by the World Bank. value terms, their production is more
Pair 2 is incorrectly matched: labour intensive as compared to
Composite Water Management Index is major minerals. Thus, they can be a
brought out by the NITI Aayog. source of large-scale employment
Pair 3 is correctly matched: generation. Hence, statement 2 is
Global Competitiveness Index is incorrect.
released by the World Economic Forum. Source: Strategy for New India @ 75
Pair 4 is incorrectly matched: by NITI Aayog; Chapter 11: minerals

Prelim IAS Test Series (2019) – GS Test 15 (31.03.2019)


22
Economic Survey, Budget, India Year Book, Social Schemes
recommends “implementation and
41. „Brasilia Declaration‟ is often seen enforcement of policies and
in the news. It is regarding which of measures to protect and promote, in
the following issues? an active way, pedestrian safety and
(a) Sustainable cities cyclists‟ mobility, such as sidewalks,
(b) Road accidents bike paths and bike lanes, proper
(c) Education lighting, radars with cameras,
(d) Biomedical waste signalling and road marking”.
Source: Strategy for New India @ 75
Answer: (b) by NITI Aayog; chapter 13: surface
Explanation: transport
Brasilia Declaration:
 The Brasilia Declaration commits to 42. The term „Mission Hundred‟ is seen
reduce the deaths caused due to in news sometimes. It is related to
traffic accidents. which of the following?
 Participants from across the world (a) Renewable energy
committed to improve road safety as (b) Railway freight terminals
part of the Brasilia Declaration, which (c) Climate change
called out for stringent traffic laws to (d) Gender parity
prevent accidents.
 The World Health Organization's Answer: (b)
(WHO) global status report on road Explanation:
safety published this year states that Mission Hundred: This mission will
1.25 million people die due to road commission at least a hundred sidings in
accidents every year, making it a the next 2 years. NITI has suggested
global health hazard. It is also the completion by 2022-23 of the
leading cause of death among people commissioning of the remaining 55 of the
aged between 15 and 29. 100 new freight terminals announced in
 The Brasilia Declaration, adopted at the Rail Budget of 2016-17 under
the second global high-level „Mission Hundred‟.
conference on road safety held in Source: Strategy for New India @ 75
Brazil, lays down recommendations by NITI Aayog; chapter 14: railways
on strengthening existing legislations,
adopting sustainable transport and 43. To make logistics in India more
strengthening post-crash response. efficient, NITI Aayog's “Strategy
In the declaration, participants for New India @ 75” Report has
reasserted their commitment to proposed to:
reduce the deaths caused due to (1) Reduce the logistics cost to
traffic accidents to half by the year less than 10 per cent of GDP
2020. from the current level of 14
 This target was set under the 2030 per cent.
Agenda for Sustainable (2) Reduce border compliance
Development. Citing its importance time to 24 hours for exports
for traffic safety, and to 48 hours for imports
the declaration promotes sustainable by 2020.
commuting ways and prioritizes Which of the statements given
pedestrians, cyclists and motorists. It above is/are correct?

Prelim IAS Test Series (2019) – GS Test 15 (31.03.2019)


23
Economic Survey, Budget, India Year Book, Social Schemes
(a) 1 only (b) 2 only
(b) 2 only (c) Both 1 and 2
(c) Both 1 and 2 (d) Neither 1 nor 2
(d) Neither 1 nor 2
Answer: (a)
Answer: (c) Explanation:
Explanation: Constraints facing the water resource
The Indian logistics industry employs managements of India:
more than 22 million people (as of 2016). Statement 1 is correct: The Easement
Between 2011-12 and 2015-16, the Act, 1882, grants groundwater ownership
logistics sector‟s value has grown at a rights, making it difficult to manage
compound annual growth rate (CAGR) of groundwater resource sustainably.
7.8 per cent. However, existing logistics Statement 2 is not correct: As per the
costs in India are high relative to other 2011 Census, only 30.8 per cent of the
countries. total rural households and 70.6 per cent
Statement 1 is correct: One of the of the total urban households get piped
objectives listed in the Report is to water supply.
reduce the logistics cost to less than 10 Source: Strategy for New India @ 75
per cent of GDP from the current level of by NITI Aayog; chapter 21: water
14 per cent. resources
Statement 2 is correct: As per the
approved National Trade Facilitation 45. In the context of inclusion, NITI
Action Plan, the target is to reduce Aayog‟s “Strategy for New India @
border compliance time to 24 hours for 75” recommends:
exports and to 48 hours for imports by (1) Government spending on
2020. education as a whole (not
Source: Strategy for New India @ 75 just school education) should
by NITI Aayog; chapter 17: logistics be increased to at least 6 per
cent of GDP by 2022.
44. With respect to water resources in (2) Government should make all
India, consider the following training compliant with the
statements: National Skills Qualification
(1) The Easement Act, 1882, Framework (NSQF).
which grants groundwater Which of the statements given
ownership rights to the above is/are correct?
landowner is one of the (a) 1 only
reasons for water over-use (b) 2 only
and depletion of groundwater (c) Both 1 and 2
levels. (d) Neither 1 nor 2
(2) Only 70 per cent of the total
rural households and 30 per Answer: (c)
cent of the total urban Explanation:
households get piped water Statement 1 is correct:
supply. Government spending on education as a
Which of the statements given whole (not just school education) should
above is/are correct? be increased to at least 6 per cent of
(a) 1 only GDP by 2022. At present, allocations to

Prelim IAS Test Series (2019) – GS Test 15 (31.03.2019)


24
Economic Survey, Budget, India Year Book, Social Schemes
the education sector by the Centre and over 22,500 soldiers who laid down their
the States remains close to 3 per cent of lives post-Independence.
GDP, while according to the World Bank, Following are some information about the
the world average in this regard is 4.7 Memorial:
per cent of GDP.  The National War Memorial draws
Statement 2 is correct: inspiration from the 'Chakravyuh'
India‟s skill development infrastructure formation.
should be brought on par with global  The Memorial is spread over 40
standards by: acres in the India Gate complex
 Developing internationally compliant behind the canopy.
National Occupation Standards  The names of 25,942 war casualties
(NOS) and the Qualification Packs have been inscribed on 16 walls of
(QP) that define a job role. the National War Memorial.
 Making all training compliant with the  The main structure has been built in
National Skills Qualification the form of four concentric circles,
Framework (NSQF). each chakra signifying different
Source: Strategy for New India @ 75 values of the armed forces.
by NITI Aayog; chapters 24 and 26 in  The Amar Chakra (circle of
inclusion section (higher education immortality): The 'Amar Chakra'
and skill development) comprises a 15-metre tall obelisk and
the eternal flame.
46. Consider the following statements  The Veerta Chakra (circle of
regarding the architecture of the bravery): The 'Veerta Chakra' is
recently inaugurated National War about the six important battles of the
Memorial in India: army, air force and navy, which have
(1) The architecture draws been depicted in bronze.
inspiration from  The Tyag Chakra (circle of sacrifice):
“Chakravyuh”, an ancient The 'Tyag Chakra' holds the names
Indian war formation. of about 25,700 battle casualties
(2) The main structure has been which have been written on a 1.5
built in the form of four metre wall.
concentric circles, each  The Rakshak Chakra (circle of
chakra signifying different protection): The outermost tier is the
values of the armed forces. 'Suraksha Chakra', comprising 695
Which of the statements given trees depicting 'standing soldiers
above is/are correct? guarding.
(a) 1 only  Adjacent to the main complex lies a
(b) 2 only tribute to the 21 Param Vir Chakra
(c) Both 1 and 2 holders with a bronze bust.
(d) Neither 1 nor 2
 The entire stonework of the
memorial, was done by Star
Answer: (c)
Mercantile of Kishore Kapur,
Explanation:
including the obelisk.
The Central Government had approved
Source:
the project for building a National War
https://www.pmindia.gov.in/en/news_
Memorial and a National War Museum
near India Gate in 2015 in memory of

Prelim IAS Test Series (2019) – GS Test 15 (31.03.2019)


25
Economic Survey, Budget, India Year Book, Social Schemes
updates/pms-mann-ki-baat- Drishyakala - Museum: Prime Minister
programme-on-all-india-radio-21/ had a look at the Exhibition on Indian Art
https://www.indiatoday.in/information/ at the Drishyakala - Museum organized
story/everything-you-need-to-know- at the same venue.
about-the-national-war-memorial-in- All four museums have been named as
delhi-1464404-2019-02-25 Kranti Mandir. This complex includes
Museums on Netaji Subhas Chandra
47. Recently four museums at Red Bose and the Indian National Army,
Fort, New Delhi were named as Yaad-e-Jallian Museum (on Jallianwala
“Kranti Mandir”. Which of the Bagh and World War 1), Museum on
following museums are included in 1857 - India‟s First War of Independence
it? and Drishyakala Museum on Indian Art,
(1) Yaad-e-Jallian Museum spanning three centuries, with over 450
(2) Museum on 1857 works of art.
(3) Drishyakala Museum Value Addition -
(4) Museum on Subhash Union Minister Mahesh Sharma in
Chandra Bose March, 2019 inaugurated „Azaadi Ke
Select the correct answer using the Diwane‟ museum, dedicated to the
code given below: unsung heroes of the country‟s freedom
(a) 1 and 2 only struggle, on the Red Fort premises here
(b) 2 and 4 only and said it would inspire the younger
(c) 1, 2 and 4 only generation. Built by the Archaeological
(d) 1, 2, 3 and 4 Survey of India (ASI), the museum is the
fifth in a series of recently inaugurated
Answer: (d) ones - the others being the Subhash
Explanation: Chandra Bose and INA museum, the
The Prime Minister, Shri Narendra Modi Yaad-e-Jallian museum, the Museum on
inaugurated the Subhas Chandra Bose 1857 (on the country‟s first war of
museum at Red Fort on January 23, Independence) the Drishyakala (a
2019, to mark his 122nd birth museum on Indian Art) - on the sacrifices
anniversary. He visited the Museum on of India‟s freedom fighters. “This new
Netaji Subhas Chandra Bose and Indian museum „Azaadi ke Diwane‟ is started in
National Army. the series of Kranti Mandir to inspire the
Yaad-e-Jallian Museum - He also younger generation and let them know
visited Yaad-e-Jallian Museum. The the cost of freedom that the great fighters
museum intends to take visitors through achieve by sacrificing their lives. I
the history of Jallianwala Bagh massacre congratulate the team of the ASI for
of 1919 and sacrifices made by Indian coming up with this digitised and
soldiers during World War 1. interactive museum,” said the culture
Museum on 1857: He also visited the Minister. The museum is a modern,
Museum on 1857 - India‟s first war of state-of-the-art, informative and
Independence and saw the portrayal of educational exhibition involving multi-
the historical narrative of the 1857 war of sensory technology to engage the
independence, showcasing the valour visitors.
and sacrifices made by Indians during Source -
the period. https://www.pmindia.gov.in/en/news_

Prelim IAS Test Series (2019) – GS Test 15 (31.03.2019)


26
Economic Survey, Budget, India Year Book, Social Schemes
updates/pms-mann-ki-baat- (c) Kerala
programme-on-all-india-radio-21/ (d) Gujarat

48. Every year in 27th October, „Infantry Answer: (b)


Day” is celebrated in India. Which Explanation:
of the following is the reason for India‟s “100 per cent organic state”
commemoration of this day as Sikkim has won the “Oscar for best
Infantry Day? policies”, conferred by the Food and
(a) On this day India won 1965 Agriculture Organisation for the world‟s
war against Pakistan. best policies promoting agro-ecological
(b) On this day India‟s first Field and sustainable food systems. Sikkim
Marshal Sam Manekshaw won the Future Policy Award 2018,
was born. beating 51 nominated policies from 25
(c) On this day Indian Army countries, according to a statement.
landed in Kashmir to fight Policies from Brazil, Denmark and Quito
Pakistani Raiders in 1947. (Ecuador) bagged silver awards. The
(d) On this day Indian Army award is co-organised by the Food and
command was transferred to Agriculture Organisation of the United
an Indian General. Nations (FAO), the World Future Council
(WFC) and IFOAM - Organics
Answer: (c) International. Sikkim became the first
Explanation: state in India to officially announce
Indian Army‟s Infantry celebrates 27 adoption of organic farming in the year
October each year as the Infantry Day to 2003 to ensure long term sustenance of
commemorate the first Infantry action soil fertility, protection of environment
post-independence. In response to the and ecology, healthy living and
threat posed to the sovereignty and decreasing the risk of health ailments. In
territorial integrity of our Nation, on this 2003, Sikkim stopped imports of
day in 1947, the leading elements of the chemical fertilizers in the State and since
Indian Army from 1st Battalion of the then the cultivatable land there is
Sikh Regiment, air landed at Srinagar. practically organic and farmers of Sikkim
This bold action by the Indian Army and are traditional users of organic manure.
resolve displayed by the Infantrymen Source:
reversed the tide of events and thwarted https://www.pmindia.gov.in/en/news_
the nefarious designs of Pakistani updates/pms-mann-ki-baat-
Raiders aided by the Pakistan Army. programme-on-all-india-radio-18/
Source: http://www.fao.org/india/news/detail-
https://www.pmindia.gov.in/en/news_ events/en/c/1157760/
updates/pms-mann-ki-baat-
programme-on-all-india-radio-18/ 50. Recently Kumbh Mela, the largest
public gathering and collective act
49. Which Indian State won the Future of faith, anywhere in the world,
Policy Gold Award (Gold Prize) concluded at Prayagraj, Uttar
2018, also nicknamed as “Oscar Pradesh. What was the tagline of
for best policies”? Kumbh Mela 2019?
(a) Arunachal Pradesh (a) Sarva Siddhi Pradah Kumbh
(b) Sikkim (b) Sarva Sukhi Pradah Kumbh

Prelim IAS Test Series (2019) – GS Test 15 (31.03.2019)


27
Economic Survey, Budget, India Year Book, Social Schemes
(c) Yadaham Jeevami, Kumbh Explanation:
Shanshe The Indian e-commerce industry has
(d) Sarva Gyanam Kumbh been on an upward growth trajectory and
Vidyate is expected to surpass the US to become
the second largest e-commerce market
Answer: (a) in the world by 2034. The E-commerce
Explanation: market is expected to reach US$ 200
Prayagraj Ardh Kumbh Mela, 2019 was billion by 2026 from US$ 38.5 billion in
the Ardh 2017.
Kumbh Mela The government has issued Guidelines
held at Triveni for FDI on E-commerce sector. 100%
Sangam in FDI under automatic route is now
Allahabad, permitted in marketplace model of e-
Uttar Pradesh, commerce. The Department of Industrial
India from 15 Policy and Promotion (DIPP) has ruled
January to 4 out foreign direct investment (FDI) in
March 2019. inventory-based e-commerce. If an e-
The logo of commerce marketplace entity exercises
2019 Kumbh shows a group of Sadhus control over inventory, it will be deemed
bathing in the Sangam confluence of the to be an inventory-based model and
Ganga and the Yamuna with a tagline cannot receive FDI.
“Sarva Siddhi Pradah Kumbh” which Value Addition:
means Kumbh is the provider of all The Department of Commerce had in its
accomplishments. recent draft E-commerce policy mooted
Value Addition - up to 49% FDI in inventory-based e-
Kumbh has been inscribed on the commerce for 100% locally made goods
UNESCO's Representative List of for majority Indian-owned and Indian-
Intangible Cultural Heritage of Humanity. controlled e-commerce ventures.
Source: India Year Book 2018 Chapter
51. Which of the following statements 19: Industry - Page 327
is correct about „Foreign Direct Vajiram and Ravi Institute – Current
Investment (FDI)‟ in the Indian E- Affairs – Feb 2019, Page no. 68
commerce sector?
(a) 100 percent FDI under 52. Consider the following statements
automatic route is permitted about „Lalit Kala Akademi‟:
in marketplace model. (1) It is an autonomous body
(b) 100 percent FDI under which is fully funded by the
automatic route is permitted Ministry of Culture.
in inventory-based model. (2) It is the Indian Government‟s
(c) Up to 51 percent FDI under apex cultural body in the field
automatic route is permitted of visual arts in India.
in marketplace model. Which of the statements given
(d) Up to 49 percent FDI under above is/are correct?
automatic route is permitted (a) 1 only
in inventory-based model. (b) 2 only
(c) Both 1 and 2
Answer: (a) (d) Neither 1 nor 2

Prelim IAS Test Series (2019) – GS Test 15 (31.03.2019)


28
Economic Survey, Budget, India Year Book, Social Schemes
India‟s total overseas trade by volume is
Answer: (c) through shipping. The security of trade
Explanation: and commerce is, therefore, of utmost
Lalit Kala Akademi, the National
Academy of Art, was set up in 1954. The
Akademi is an institution that has
rendered service to the nation in the art‟s
sphere long before the world woke up to
the global impact of Indian art.
Statement (1) is correct. Lalit Kala
Akademi is an autonomous body, which
is fully funded by the Ministry of Culture.
Statement (2) is correct. It is the primacy and importance for India. The
government‟s apex cultural body in the location of choke points such as the
field of visual arts in India. Straits of Hormuz, Malacca and others
Value Addition: and the presence of multinational
Lalit Kala Akademi has exhibitions and maritime forces make the scenario in the
educational programmes of unparalleled Indian Ocean Region (IOR) dynamic.
significance; sustains a library, collection, The Strait of Hormuz is a strait between
archives, conservation laboratory and the Persian Gulf and the Gulf of Oman.
supports scholars and publications of Source: India Year Book 2018 Chapter
pre-eminent intellectual merit all over the 9: Defence Page no. 119
country. India Year Book 2019, Chapter 9:
Source: India Year Book 2018 Chapter Defence Page no. 127
5: Culture and Tourism – Page no. 59
India Year Book 2019, Chapter 5: 54. The Padma Awards are announced
Culture and Tourism, Page no. 59 on the eve of the Indian Republic
Day every year. For which among
53. Strait of Hormuz is a strait between the following disciplines/fields of
which of the following two water activities such awards are given?
bodies? (1) Literature
(a) Persian Gulf and Gulf of (2) Civil Service
Oman (3) Sports
(b) Red Sea and Gulf of Aden (4) Medicine
(c) Arabian Sea and Gulf of Select the correct answer using the
Aden code given below:
(d) Bay of Bengal and Indian (a) 1 and 3 only
Ocean (b) 1, 2 and 3 only
(c) 1, 2 and 4 only
Answer: (a) (d) 1, 2, 3 and 4
Explanation:
India‟s central peninsular position in the Answer: (d)
Indian Ocean and geo-political Explanation:
circumstances makes this country reliant Padma Awards - one of the highest
on the seas. India‟s oil imports have risen civilian awards of the country, are
to nearly 80 per cent, almost all of which conferred in three categories, namely,
are transported by sea. 95 per cent of Padma Vibhushan, Padma Bhushan and

Prelim IAS Test Series (2019) – GS Test 15 (31.03.2019)


29
Economic Survey, Budget, India Year Book, Social Schemes
Padma Shri. The Awards are given in students for pursuing technical education
various disciplines/ fields of activities, in AICTE approved institution. This is to
viz.- art, social work, public affairs, help them to achieve their college goals,
science and engineering, trade and despite learning issues, environmental
industry, medicine, literature and challenges or medical bills.
education, sports, civil service, etc. Saksham is a Ministry of Human
„Padma Vibhushan‟ is awarded for Resource Development (MHRD)
exceptional and distinguished service; Scheme. This is an attempt to give every
„Padma Bhushan‟ for distinguished young student, who is otherwise specially
service of high order and „Padma Shri‟ abled, the opportunity to study further
for distinguished service in any field. and prepare for a successful future.
Value Addition: Source: India Year Book 2018 Chapter
These awards are conferred by the 10: Education - Page no. 157
President of India at ceremonial functions India Year Book 2019, Chapter 10:
which are held at Rashtrapati Bhawan Education, Page no. 168
usually around March/ April every year.
This year the President of India has 56. Which of the following statements
approved conferment of 112 Padma is correct about the „Wildlife Crime
Awards. Control Bureau‟ of India?
Source: India Year Book 2018 Chapter (a) It is a constitutional body.
32 General Information – Page no. 635 (b) It has been established by a
India Year Book 2019, Chapter 32: Cabinet resolution.
General Information, Page no. 689 (c) It is a statutory body.
(d) It is a non-profit autonomous
55. With reference to welfare schemes organization.
for vulnerable sections, what is
„SAKSHAM‟? Answer: (c)
(a) Scholarship for differently- Explanation:
abled children. The Wildlife Crime Control Bureau,
(b) Scholarship for vocational constituted in 2007 is a statutory multi-
studies for minorities. disciplinary body under the Wildlife
(c) Scholarship for higher Protection Act, 1972. It is established
education of girls. under the Ministry of Environment, Forest
(d) Scheme for providing quality and Climate Change, to combat
education in madrasas. organized wildlife crime in the country.
The Bureau has its headquarters in New
Answer: (a) Delhi and five regional offices at Delhi,
Explanation: Kolkata, Mumbai, Chennai and Jabalpur.
SAKSHAM - Scholarship for Differently- It is mandated to collect and collate
Abled Children, a scheme of „All India intelligence related to organized wildlife
Council for Technical Education (AICTE)‟ crime activities and to disseminate the
aims to provide encouragement and same to state and other enforcement
support to differently-abled children for agencies for immediate action so as to
pursuing technical education. apprehend the criminals; to establish a
Scholarships amounting to ₹ 5 crore per centralized wildlife crime databank;
annum as tuition fees and incidentals are coordinate actions by various agencies in
to be provided to needy and meritorious connection with the enforcement of the

Prelim IAS Test Series (2019) – GS Test 15 (31.03.2019)


30
Economic Survey, Budget, India Year Book, Social Schemes
provisions of the Wildlife Protection Act plants, interception and diversion of
1972; assist foreign authorities and sewage, desilting, catchment area
international organization concerned to treatment, storm water management, etc.
facilitate co-ordination and universal Conservation works for 34 lakes have
action for wildlife crime control; capacity been completed. Major projects presently
building of the wildlife crime enforcement under implementation include Dal Lake in
agencies for scientific and professional Jammu & Kashmir, Shivpuri and Sindh
investigation into wildlife crime and assist Sagar lakes in Madhya Pradesh, twin
state governments to ensure success in lakes at Mokokchung in Nagaland (NE
prosecutions related to wildlife crimes; region), Annasagar, Pushkar and Pichola
and advise the Government of India on lake in Rajasthan, Ramgarh Tal and
issues relating to wildlife crimes having Laxmi Tal in Uttar Pradesh.
national and international ramifications, Source: India Year Book 2018 Chapter
relevant policy and laws. 12: Environment Page no. 187
Source: India Year Book 2018 Chapter India Year Book 2019, Chapter 12:
12 Environment Page no. 179 Environment, Page no. 197
India Year Book 2019, Chapter 12:
Environment, Page no. 188 58. Consider the following statements
about the Global Environment
57. Which of the following pairs is/are Facility (GEF):
correctly matched? (1) India is a founder member of
Lake State the GEF.
1. Sindh : Maharashtra (2) The GEF was established on
Sagar the eve of the 1992 Rio Earth
2. Ramgarh : Uttar Summit.
Tal Pradesh (3) India is both a donor to and a
3. Pushkar : Rajasthan recipient of the GEF.
Select the correct answer using the Which of the statements given
code given below: above is/are correct?
(a) 2 only (a) 1 and 2 only
(b) 1 and 2 only (b) 3 only
(c) 2 and 3 only (c) 2 and 3 only
(d) 1, 2 and 3 (d) 1, 2 and 3

Answer: (c) Answer: (d)


Explanation: Explanation:
The correct pairs are: The Global Environment Facility - GEF is
1. Sindh Sagar- Madhya Pradesh multilateral financial mechanism that
2. Ramgarh Tal - Uttar Pradesh provides grants to developing countries
3. Pushkar - Rajasthan for projects that benefit global
So far under the National Lake environment and promote sustainable
Conservation Plan, a total of 46 projects development.
for conservation of 63 lakes in India have Statement 1 is correct: India is a
been sanctioned in 14 States for founder member of Global Environment
undertaking works like providing Facility (GEF). With India‟s increased
sewerage system and sewage treatment financing commitment, it is playing
crucial role to mitigate climate change

Prelim IAS Test Series (2019) – GS Test 15 (31.03.2019)


31
Economic Survey, Budget, India Year Book, Social Schemes
and other environmental challenges Price (SMP) of sugarcane was replaced
across the globe. India is one of the most with the „Fair and Remunerative Price
vulnerable countries to the impact of (FRP).
climate change. The sugarcane price is announced by the
Statement 2 is correct. The Global Central Government on the basis of the
Environment Facility (GEF), established recommendations of the Commission for
on the eve of the 1992 Rio Earth Summit, Agricultural Costs and Prices (CACP)
is a catalyst for action on the after consulting the State governments
environment - and much more. Through and associations of sugar industry. The
its strategic investments, the GEF works amendment of the Sugarcane (Control)
with partners to tackle the planet's Order, 1966 provides for fixation of FRP
biggest environmental issues. The GEF of sugarcane based on the following
mandate is decided as per the guidance factors:-
provided by the Conference of the parties (a) cost of production of sugarcane;
of the multilateral environmental (b) return to the growers from alternative
conventions namely Convention on crops and the general trend of prices
Biological Diversity (CBD), United of agricultural commodities;
Nations Framework Convention on (c) availability of sugar consumers at a
Climate Change (UNFCCC), United fair price;
Nations Convention to Combat (d) price at which sugar produced from
Desertification (UNCCD), Stockholm sugarcane is sold;
Convention on PoPs and Minamata (e) reasonable margins.
Convention on Mercury. Source: India Year Book 2018 Chapter
Statement 3 is correct. India has been 15: Food and Civil Supplies Page no.
a leading developing country participant 280
in the GEF since its inception and has India Year Book 2019, Chapter 15:
played a major role in shaping the GEF. Food, Civil Supplies and Consumer
India is both a donor to and a recipient of Affairs, Page no. 272
the GEF.
Source: India Year Book 2018 Chapter 60. For which of the following entities,
12 Environment Page no. 194 the Department of Economic
India Year Book 2019, Chapter 12: Affairs prepares Annual Budget so
Environment, Page no. 206 as to get them presented before
the Indian Parliament?
59. The Term „Fair and Remunerative (1) The Union Government
Price‟ is associated with which of (2) A State Government under
the following commodities? the President‟s Rule
(a) Wheat (3) A Union Territory under
(b) Sugarcane central administration
(c) Rice Select the correct answer using the
(d) Pulses code given below:
(a) 1 and 2 only
Answer: (b) (b) 1 and 3 only
Explanation: (c) 2 and 3 only
In India, with the amendment of the (d) 1, 2 and 3
Sugarcane (Control) Order, 1966 in
2009, the concept of Statutory Minimum Answer: (d)

Prelim IAS Test Series (2019) – GS Test 15 (31.03.2019)


32
Economic Survey, Budget, India Year Book, Social Schemes
Explanation: Answer: (c)
The Department of Economic Affairs, Explanation:
Ministry of Finance, is responsible for The Members of Parliament Local Area
advice on economic issues having a Development Scheme (MPLADS) was
bearing on internal and external aspects launched in 1993 for creation of durable
of the Indian economy including inflation, community assets and for provision of
price control, foreign exchange basic facilities including community
management, Official Development infrastructure, based on locally felt needs
Assistance, bilateral and multilateral to be taken up in their
engagement with international financial constituencies/states.
institutions and other countries. The Statement (1) is incorrect. Initially, the
Department of Economic Affairs is MPLADS was under the control of the
responsible for preparation and Ministry of Rural Development. The
presentation of Union Budget to the subject relating to the MPLADS was
Parliament and the Budget for the State transferred to the Ministry of Statistics
governments under President‟s Rule and and Programme Implementation in 1994.
Union territory administrations. The Ministry of Statistics and Programme
Source: India Year Book 2018 Chapter Implementation (MSPI) is the nodal
13: Finance Page no. 195 ministry to coordinate work under the
India Year Book 2019, Chapter 13: scheme. The scheme is governed by a
Finance, Page no. 207 set of guidelines, which have been
comprehensively revised from time to
61. The Members of Parliament Local time. The salient features of the MPLAD
Area Development Scheme Scheme include:
(MPLADS) was launched in 1993  MPLADS is a Central Plan Scheme
to provide a mechanism for the fully funded by the Government of
Members of Indian Parliament to India under which fund is released in
recommend works of the form of grants-in-aid directly to
developmental nature. Consider the district authorities.
the following statements with  The funds released under the
respect to MPLADS: Scheme are non-lapsable, i.e. the
(1) It is supervised by the entitlement of funds not released in a
Ministry of Rural particular year, is carried forward to
Development. the subsequent years, subject to
(2) The funds sanctioned under eligibility. At present, the annual
it are non-lapsable. entitlement per Member of the
(3) The role of the Members of Parliament / constituency is ₹ 5 crore.
Parliament is only to Hence, statement (2) is correct.
recommend works and not to  Under MPLADS, the role of the
execute it. Members of Parliament is limited to
Which of the statements given recommend works. Thereafter, it is
above are correct? the responsibility of the district
(a) 1 and 2 only authority to sanction, execute and
(b) 1 and 3 only complete the works recommended by
(c) 2 and 3 only the Members of Parliament within the
(d) 1, 2 and 3 stipulated time period. Hence,
statement (3) is correct.

Prelim IAS Test Series (2019) – GS Test 15 (31.03.2019)


33
Economic Survey, Budget, India Year Book, Social Schemes
Source: India Year Book 2018 Chapter (a) The motto „Satyameva
6: Basic Economic Data Page no. 84 Jayate‟, has been taken from
India Year Book 2019, Chapter 6: Aitreya Upanishad.
Basic Economic Data, Page no. 88 (b) National Anthem was first
sung in 1950.
62. Which of the following institutions (c) National Song was first sung
are implementing „Kisan Credit in 1896.
Card‟ scheme? (d) Dates of National calendar do
(1) Commercial Banks not have a permanent
(2) Cooperative Banks correspondence with
(3) Regional Rural Banks Gregorian calendar.
Select the correct answer using the
code given below: Answer: (c)
(a) 1 and 2 only Explanation:
(b) 1 and 3 only Statement 1 is incorrect. The motto
(c) 2 and 3 only “Satyameva Jayate” - Truth alone
(d) 1, 2 and 3 triumphs, has been taken from Mundaka
Upanishad.
Answer: (d) Statement 2 is incorrect. National
Explanation: anthem was first sung in 1911 Kolkata
The Kisan Credit Card (KCC) scheme session of Indian National Congress.
has helped in augmenting credit flow for Statement 3 is correct. National song
agricultural activities. The scope of the was first sung in the 1896 session of
KCC has been broad-based to include Indian National Congress.
term credit and consumption needs. Statement 4 is incorrect. Dates of
The Kisan Credit Card Scheme is in National calendar do have a permanent
operation throughout the country and is correspondence with Gregorian calendar.
implemented by Commercial Banks, Source: India Year Book 2018 Chapter
Cooperative Banks and Regional Rural 2: National Symbols Page no. 17-19
Banks (RRB). India Year Book 2019, Chapter 2:
The KCC scheme has been simplified National Symbols, Page no. 9 - 11
and converted into ATM enabled debit
card with, inter-alia, facilities of one-time 64. Which of the following is/are
documentation, built-in cost escalation in powers assigned to the President
the limit, any number of withdrawals of India?
within the limit, etc. which eliminates the (1) The President makes
need for disbursement through camps recommendations for
and mitigates the vulnerability of farmers introducing financial and
to middlemen. money bills.
Source: India Year Book 2018 Chapter (2) The supreme command of
4: Agriculture Page 51 the Indian defence forces
Vajiram and Ravi Institute, Current vests in the President.
Affairs – January 2019, Page no. 65 (3) The President is ex-officio
Chairman of the Rajya
63. Which of the following statements Sabha.
is correct about „Indian National Select the correct answer using the
Symbols‟? code below:

Prelim IAS Test Series (2019) – GS Test 15 (31.03.2019)


34
Economic Survey, Budget, India Year Book, Social Schemes
(a) 1 only (1) The average elevation of the
(b) 1 and 2 only Western Ghats in India is
(c) 2 and 3 only greater than that of the
(d) 1, 2 and 3 Eastern Ghats.
(2) Between the Western Ghats
Answer: (b) and the Arabian Sea lies a
Explanation: broader coastal area, while
The President of India is elected by the between the Eastern Ghats
members of an electoral college and the Bay of Bengal there
consisting of elected members of both is a narrow coastal area.
the Houses of Parliament and Legislative Which of the statements given
Assemblies of the States in accordance above is/are correct?
with the system of proportional (a) 1 only
representation by means of single (b) 2 only
transferable vote. (c) Both 1 and 2
Statement (1) is correct: The President (d) Neither 1 nor 2
summons, prorogues, addresses, sends
messages to the Parliament and Answer: (a)
dissolves the Lok Sabha; promulgates Explanation:
ordinances at any time, except when Statement 1 is correct: The Peninsula
both Houses of the Parliament are in is flanked on the one side by the Eastern
session; makes recommendations for Ghats (where average elevation is about
introducing financial and money bills and 610 metres) and on the other by the
gives assent to bills; grants pardons, Western Ghats (where average elevation
reprieves, respites or remission of generally varies from 915 to 1,220
punishment or suspends, remits or metres, rising in places to over 2,440
commutes sentences in certain cases. metres.)
Statement (2) is correct: The executive Statement 2 is incorrect: Between the
power of the Union is vested in the Western Ghats and the Arabian Sea lies
President and is exercised by him either a narrower coastal area, while between
directly or through officers subordinate to the Eastern Ghats and the Bay of Bengal
him in accordance with the Constitution there is a broader coastal area. On the
of India. The supreme command of the western side, the average width of coast
defence forces of the Union also vests in is 65 km while on the eastern side it is
him. 120 km. The western coastal plains run
Statement (3) is incorrect: The Vice from Kutchch to Kanyakumari while the
President of India is the ex-officio eastern plains run from Subarnarekha
Chairman of the Rajya Sabha, who river to Kanyakumari.
presides over its sessions. Source: India Year Book 2018 Chapter
Source: India Year Book 2018 Chapter 1 Land and people Page no. 9
3 Polity Page no. 22 India Year Book 2019, Chapter 1: Land
India Year Book 2019, Chapter 3: and People, Page no. 2
Polity, Page no. 14
66. The Government of India
65. Consider the following statements announces minimum support
with respect to Peninsular India: prices (MSPs) on the basis of
recommendations of the

Prelim IAS Test Series (2019) – GS Test 15 (31.03.2019)


35
Economic Survey, Budget, India Year Book, Social Schemes
Commission for Agricultural Costs (1) to celebrate the unity in
and Prices (CACP). CACP is diversity of India
mandated to advice on the price (2) to promote the spirit of
policy (MSP) of which of the national integration in India
following crops? (3) to participate in bilateral
(1) Pulses military exercises
(2) Oilseeds (4) to showcase India‟s rich
(3) Raw Jute heritage and culture
Select the correct answer using the Select the correct answer using the
code given below: code given below:
(a) 1 only (a) 1 and 2 only
(b) 1 and 2 only (b) 1, 2 and 3 only
(c) 2 and 3 only (c) 1, 2 and 4 only
(d) 1, 2 and 3 (d) 2, 3 and 4 only

Answer: (d) Answer: (c)


Explanation: Explanation: „Ek Bharat Shreshtha
The Commission for Agricultural Costs Bharat‟ was announced in 2015 on the
and Prices (CACP) was set up with a occasion of the 140th Birth Anniversary
view to evolve a balanced and integrated of Sardar Vallabhbhai Patel. The broad
price structure. It is mandated to advice objectives of the initiative are:-
on the price policy (MSP) of certain  to celebrate the unity in diversity of
crops. These include seven cereal crops India and to maintain and strengthen
(paddy, wheat, jowar, bajra, maize, ragi the fabric of traditionally existing
and barley), five pulse crops (gram, tur, emotional bonds between the people
moong, urad and lentil), seven oilseeds of India. Hence, point (1) is correct.
(groundnut, sunflower seed, soyabean,  to promote the spirit of national
rapeseed, mustard, safflower, nigerseed integration through a deep and
and sesamum), copra (dried coconut), structured engagement between all
cotton, raw jute and sugarcane. Indian States and Union Territories
The CACP submits its recommendations through a year-long planned
in the form of Price Policy Reports every engagement between the States.
year, separately for five groups of Hence, point (2) is correct.
commodities namely kharif crops, rabi  to showcase the rich heritage and
crops, sugarcane, raw jute and copra. culture, customs and traditions of
Source: India Year Book 2018 Chapter Indian states for enabling people to
4 Agriculture Page no. 52 understand and appreciate the
India Year Book 2019, Chapter 4: diversity that is India, thus fostering a
Agriculture, Page no. 51-52 sense of common identity. Hence,
point (4) is correct.
67. „Ek Bharat Shreshtha Bharat‟  to establish long-term engagements;
programme was announced in  to create an environment which
2015 on the occasion of the 140th promotes learning between States by
birth anniversary of Sardar sharing best practices and
Vallabhbhai Patel. Which among experiences.
the following are its broad To participate in bilateral military exercise
objectives? is not an objective under the „Ek Bharat

Prelim IAS Test Series (2019) – GS Test 15 (31.03.2019)


36
Economic Survey, Budget, India Year Book, Social Schemes
Shreshtha Bharat‟. Hence, point (3) is not Negrito tribe) in the Nicobar Group of
incorrect. Islands. Hence, statement (2) is
Source: India Year Book 2018 Chapter incorrect.
5 Culture and Tourism Page no. 64 Source: India Year Book 2018 Chapter
India Year Book 2019, Chapter 5: 30: States and Union Territories -
Culture and Tourism, Page no. 65 Page 607
India Year Book 2019, Chapter 30:
68. Consider the following statements States and Union Territories, Page no.
with reference to the „Andaman 658
and Nicobar Islands‟:
(1) The Islands receive rainfall 69. Which of the following pairs is/are
from both the south-west and correctly matched?
north-east monsoons. Mission Ministry
(2) The inhabitants of Islands 1. Atal : Ministry of
include Negrito tribes like Innovation Science and
Nicobarese and Shompens. Mission Technology
Which of the statements given 2. National : Ministry of
above is/are not correct? Mission for Culture
(a) 1 only Manuscripts
(b) 2 only 3. National : Ministry of
(c) Both 1 and 2 Water Rural
(d) Neither 1 nor 2 Mission Development
Select the correct answer using the
Answer: (b) code given below:
Explanation: (a) 2 only
The Union Territory of the Andaman and (b) 2 and 3 only
Nicobar Islands is situated between 6° (c) 1, 2 and 3
and 14° latitude and 92° and 94° (d) None of the above
longitude. The Islands located north of
10° north latitude are known as Answer: (a)
Andaman Group of Islands while islands Explanation:
located south of 10° north latitude are Atal Innovation Mission (AIM):
called Nicobar Group of Islands. The Government of India has established Atal
climate of the Islands can be defined as Innovation Mission (AIM) and Self
humid, tropical coastal climate. Employment and Talent Utilisation
Statement (1) is correct: The Islands (SETU) under NITI Aayog. NITI Aayog
receive rainfall from both the south-west will hire Mission Director and other
and north-east monsoons and maximum appropriate manpower. Hence, pair (1)
precipitation is between May and is incorrect. The headquarter of the
December. Mission will be in New Delhi. Atal
The original inhabitants of the Islands Innovation Mission (AIM) is the
lived in the forests on hunting and Government of India‟s endeavour to
fishing. There are four Negrito tribes, viz., promote a culture of innovation and
Great Andamanese, Onge, Jarawa and entrepreneurship. Its objective is to serve
Sentinalese in the Andaman Group of as a platform for promotion of world-class
Islands and two Mongoloid tribes, viz., Innovation Hubs, Grand Challenges,
Nicobarese and Shompens (which are

Prelim IAS Test Series (2019) – GS Test 15 (31.03.2019)


37
Economic Survey, Budget, India Year Book, Social Schemes
Start-up businesses and other self- Page no 485, Chapter 23: Planning,
employment activities, particularly in Page no. 434
technology driven areas. India Year Book 2019, Chapter 5:
National Mission for Manuscripts: The
Culture and Tourism - Page no.67;
mandate of the Ministry of Culture
revolves around the functions like Chapter 27: Water Resources, Page
preservation and conservation of ancient no 518, Chapter 23: Planning, Page
cultural heritage and promotion of art and no. 434
culture both tangible and intangible in the
country. The National Mission for 70. GPS Aided Geo Augmented
Manuscripts (NMM) was launched in Navigation (GAGAN) in India is
2003 under Ministry of Culture to reclaim being implemented jointly by which
India‟s inheritance of knowledge among the following organisations?
contained in the vast treasure of (1) Airport Authority of India
manuscripts. Hence, pair (2) is correct. (2) Defence Research and
There are Manuscripts Resource Centres Development Organization
(MRC), Manuscripts Conservation (3) Indian Space Research
Centres (MCC), Manuscripts Partner Organization
Centres (MPC) and Manuscripts (4) Bureau of Civil Aviation
Conservation Partner Centres (MCPC) Select the correct answer using the
all over India to locate, to document, to code given below:
conserve and to digitize the tangible (a) 1 and 2 only
heritage of India. (b) 1 and 3 only
National Water Mission: With a view to (c) 1, 2 and 4 only
address climate change and the related (d) 2, 3 and 4 only
issues, the National Action Plan on
Climate Change (NAPCC) laid down the Answer: (b)
principles and identified the approach to Explanation:
be adopted to meet the challenges of The GPS Aided Geo Augmented
impact of climate change through eight Navigation (GAGAN) is an augmentation
national missions. National Water system to enhance the accuracy and
Mission is one of the missions under integrity of GPS signals to meet precision
NAPCC. The main objective of the approach requirements in Civil Aviation
National Water Mission is “conservation and it is being implemented jointly by the
of water, minimizing wastage and Airport Authority of India (AAI) and the
ensuring its more equitable distribution Indian Space Research Organization
both across and within States through (ISRO). It is fully operational since May
integrated water resources development 2015 and available on 24x7 basis.
and management”. The mission is Presently GAGAN Signal-in-Space is
administered by Ministry of Water available to users from two geostationary
Resources, River Development and satellites (GSAT-8 and GSAT-10) and
Ganga Rejuvenation Hence, pair (3) is third geo-satellite GSAT-15 is in the
incorrect. process of integration with GAGAN
Source: India Year Book 2018, system.
Chapter 5: Culture and Tourism - Page Source: India Year Book 2018 Chapter
no.67; Chapter 27: Water Resources, 26: Transport Page no. 482

Prelim IAS Test Series (2019) – GS Test 15 (31.03.2019)


38
Economic Survey, Budget, India Year Book, Social Schemes
India Year Book 2019, Chapter 26, 72. Consider the following statements:
Transport, Page no. 513 (1) Urban development is a
State subject but the
71. Which among the following is a Government of India plays a
relatively more efficient coolant for coordinating and monitoring
a „Fast Breeder Nuclear Reactor‟ role.
being developed under the Indian (2) According to the Census
Nuclear Power Generation 2011, as many as 52 cities in
Programme? India had population of a
(a) Heavy Water million plus.
(b) Light Water Which of the statements given
(c) Liquid Sodium above is/are correct?
(d) None of the above (a) 1 only
(b) 2 only
Answer: (c) (c) Both 1 and 2
Explanation: (d) Neither 1 nor 2
For the second stage of the Indian
Nuclear Power Generation Programme, Answer: (c)
the Indira Gandhi Centre for Atomic Explanation:
Research (IGCAR) is pursuing The Ministry of Urban Development has
development of sodium cooled fast the responsibility of broad policy
breeder reactors and associated fuel formulation and monitoring of
cycle technologies. Using water as programmes in the area of urban
coolant would slow down the neutrons, development.
but the use of liquid sodium avoids that Statement (1) is correct. Urban
moderation and provides a very efficient development is a State subject but the
heat transfer medium. The question of Government of India plays a coordinating
safety has been raised since sodium and monitoring role and also supports
metal is an extremely reactive chemical urban development through centrally
and burns on contact with air or water sponsored schemes. The Ministry of
(sometimes explosively on contact with Urban Development addresses various
water). It is true that the liquid sodium issues of urban sector through policy
must be protected from contact with air guidelines, legislative guidance and
or water at all times, kept in a sealed sectoral programmes.
system. However, it has been found that Statement (2) is correct. Urbanization in
the safety issues are not significantly India has become an important and
greater than those with high-pressure irreversible process, and an important
water and steam in the light-water determinant of national economic growth
reactors. and poverty reduction. The process of
Source: India Year Book 2018 Chapter urbanization is characterized by a
25: Scientific and Technological dramatic increase in the number of large
Developments, Page no. 458 cities, although India may be said to be in
India Year Book 2019, Chapter 25: the midst of transition from a
Science and Technological predominantly rural to a quasi urban
Developments, Page no. 483 society. According to the Census 2011,
as many as 52 cities in India had
population of a million plus. At current

Prelim IAS Test Series (2019) – GS Test 15 (31.03.2019)


39
Economic Survey, Budget, India Year Book, Social Schemes
rate of growth, urban population in India disciplinary team under a Command
will reach a staggering total of 575 million Area Development Authority. The
by 2030 A.D programme is being implemented under
Source: India Year Book 2018 Chapter the Pradhan Mantri Krishi Sinchai Yojana
24: Rural and Urban Development, (PMKSY) - Har Khet Ko Pani - from
Page – 448 2015-16.
India Year Book 2019, Chapter 17: Source: India Year Book 2018,
Housing and Urban Affairs, Page no. Chapter 27: Water Resources, Page
302 no. 488
India Year Book 2019, Chapter 27:
73. Consider the following statements Water Resources, Page no. 521
with reference to the Command
Area Development (CAD) 74. Which of the following pairs is/are
programme in India: correctly matched?
(1) The CAD programme was India‟s With
launched in 1974-75 to Military Country
enhance water use efficiency Exercise
in irrigation. 1. Yudh Abhyas : Russia
(2) From 2015-16, the CAD 2. Mitra Shakti : Bhutan
programme is being
3. Ajeya Warrior : France
implemented under the
Select the correct answer using the
Pradhan Mantri Krishi
code given below:
Sinchai Yojana (PMKSY).
(a) 1 only
Which of the statements given
(b) 2 and 3 only
above is/are not correct?
(c) 1, 2 and 3
(a) 1 only
(d) None of the above
(b) 2 only
(c) Both 1 and 2
Answer: (d)
(d) Neither 1 nor 2
Explanation:
India‟s joint military / training exercises:
Answer: (d)
(1) Yudh Abhyas : USA
Explanation:
(2) Mitra Shakti: Sri Lanka
Statement (1) is correct: The Centrally
(3) Ajeya Warrior: U.K
Sponsored Command Area Development
Exercise Yudh Abhyas, a joint Indo-US
(CAD) programme was launched in
Military exercise. It was started in 2004
1974-75 for development of adequate
under US Army Pacific Partnership
delivery system of irrigation water up to
Programme. Exercise Yudh Abhyas
farmers‟ field with an objective to
strengthens and broadens
enhance water use efficiency and
interoperability and cooperation between
production and productivity of crops per
the Indian and US armies.
unit of land and water for improving
Mitra-Shakti is Joint Training Exercise
socio-economic condition of farmers.
between India and Sri Lanka. The
Statement (2) is correct: The
exercise is based on Counter Terrorist
programme envisages integration of all
Operations and an Infantry company
activities relating to irrigated agriculture
from both the countries participates in the
in a coordinated manner with multi-
same.

Prelim IAS Test Series (2019) – GS Test 15 (31.03.2019)


40
Economic Survey, Budget, India Year Book, Social Schemes
Ajeya Warrior, a joint exercise between Statement (2) is correct: At the Central
the armies of India and UK. The Exercise Level, Ministry of Women and Child
is held biannually in the two countries, Development (MWCD) is responsible for
alternatively. The aim of the Exercise is promoting Gender Budgeting. The
to build and promote positive military MWCD as the nodal agency for gender
relations between Indian and UK Army budgeting is undertaking several
and to enhance their ability to undertake initiatives for taking it forward at the
joint tactical level operations in Counter national and state levels. 57 central
Insurgency/Counter Terrorism ministries and departments have set up
Environment under United Nations GBCs which are expected to serve as a
Charter. focal point for coordinating gender
Source: India Year Book 2019 Chapter budgeting initiatives, both intra and inter-
9: Defence Page no. 124 ministerial. 21 States and Union
Territories have designated Gender
75. Consider the following statements: Budgeting nodal centres.
(1) Gender Budgeting (GB) is a Source: India Year Book 2018 Chapter
powerful tool for achieving 28 Welfare Page no. 527
gender mainstreaming. India Year Book 2019, Chapter 28:
(2) In India, the Ministry of Welfare, Page no. 566
Women and Child
Development is the nodal 76. Consider the following statements
agency for gender budgeting. with respect to the Krishi Kalyan
Which of the statements given Abhiyan:
above is/are correct? (1) It has been launched in all
(a) 1 only Panchayats across the
(b) 2 only country to assist and advice
(c) Both 1 and 2 farmers on ways to improve
(d) Neither 1 nor 2 the farming techniques.
(2) Under this scheme, soil
Answer: (c) health cards will be
Explanation: distributed and farmers would
Statement (1) is correct: Gender be provided demonstration
Budgeting (GB) is a powerful tool for programmes on micro-
achieving gender mainstreaming so as to irrigation.
ensure that benefits of development (3) It includes 100 percent
reach women as much as men. It is not coverage of bovine
an accounting exercise alone but an vaccination for Foot and
ongoing process of keeping a gender Mouth Disease (FMD).
perspective at various steps of budget Which of the statements given
planning, allocation, implementation, above is/are correct?
impact/outcome assessment, review and (a) 1 only
audit. To institutionalize such budgeting (b) 2 and 3 only
in the country, the setting up of Gender (c) 1 and 2 only
Budgeting Cells (GBCs) in all (d) 1, 2 and 3
Ministries/Departments was mandated by
the Ministry of Finance in 2007. Answer: (b)
Explanation:

Prelim IAS Test Series (2019) – GS Test 15 (31.03.2019)


41
Economic Survey, Budget, India Year Book, Social Schemes
Statement 1 is incorrect: The to illuminate dark regions
government has launched Krishi Kalyan across five States through
Abhiyaan to aid, assist and advice solar power.
farmers on how to improve their farming (2) The scheme is being
techniques and raise their incomes. The implemented on the basis of
Krishi Kalyan Abhiyaan will be 50:50 cost sharing between
undertaken in 25 Villages with more than the States and the Union
1000 population each in Aspirational Government.
Districts identified in consultation with Select the correct answer using the
the Ministry of Rural Development as per code given below:
the directions of NITI Aayog. (a) 1 only
Statements 2 and 3 are correct: (b) 2 only
Krishi Kalyan Abhiyaan would include: (c) Both 1 and 2
 Distribution of Soil Health Cards to (d) Neither 1 nor 2
all farmers
 100% coverage of bovine vaccination Answer: (a)
for Foot and Mouth Disease (FMD) Explanation:
in each village. Statement 1 is correct: The Ministry of
 100% coverage of Sheep and Goat New and Renewable Energy (MNRE)
for eradication of Peste des Petits launched the Atal Jyoti Yojana (AJAY)
ruminants (PPR). to illuminate dark regions across five
 Distribution of Mini Kits of pulses and States through solar power. It is a sub
oilseeds to all the beneficiaries. scheme under off–grid and decentralized
 Distribution of Horticulture/Agro solar application scheme of the Ministry
Forestry/Bamboo plant @ 5 per of New and Renewable Energy
family (location appropriate) (MNRE), Government of India.
 Making 100 NADAP/NADEP Statement 2 is incorrect: The total cost
Compost Pits in each village of the scheme is Rs 499.30 crore. The
 Artificial insemination saturation MNRE will provide 75% of the cost of
street lights and remaining 25% will
 Demonstration programmes on
come from the Member of Parliament
Micro-irrigation
Local Area Development funds
 Demonstrations of integrated
(MPLADS), Panchayat funds or
cropping practice
Municipalities and other Urban Local
Sources:
Bodies (ULBs) Funds.
 http://pib.nic.in/newsite/PrintRelea Sources:
se.aspx?relid=179747
 https://mnre.gov.in/content/launch
 https://icarrcer.in/wp- -atal-jyothi-yojana-ajay-phase-
content/uploads/Report-on-Krishi- ii437mb-pdf
Kalyan-Abhiyan.pdf
 http://vikaspedia.in/energy/policy-
support/renewable-energy-1/atal-
77. Which of the following statements
jyoti-yojana
is/are correct regarding the Atal
 http://ajay.eeslindia.org/
Jyoti Yojana?
(1) It has been launched by the
78. With reference to the Deen Dayal
Ministry of New and
Disabled Rehabilitation Scheme
Renewable Energy (MNRE)

Prelim IAS Test Series (2019) – GS Test 15 (31.03.2019)


42
Economic Survey, Budget, India Year Book, Social Schemes
(DDRS), consider the following intervention, development of daily living
statements: skills, education, skill-development
(1) It has been launched to oriented towards employability, training
create an enabling and awareness generation. With a view
environment to ensure equal to inclusion of persons with disabilities in
opportunities, equity, social the mainstream of society and actualizing
justice and empowerment of their potential, the thrust would be on
persons with disabilities. education and training programmes.
(2) Under this scheme, financial Statement 3 is correct: DDRS is a
assistance is provided to the Central Sector Scheme of Government of
voluntary organizations to India being implemented since 1999 by
provide services necessary the Department of Empowerment of
for rehabilitation of persons Persons with Disabilities, Ministry of
with disabilities. Social Justice & Empowerment.
(3) It is a Central Sector Scheme Sources:
of the Government of India  http://pib.nic.in/newsite/PrintRelea
implemented by the Ministry se.aspx?relid=187566
of Social Justice &  http://disabilityaffairs.gov.in/uploa
Empowerment, Department d/uploadfiles/files/ddrs(1).pdf
of Empowerment of Persons
with Disabilities 79. Consider the following statements
Which of the statements given with respect to UDAN 3.0:
above are correct? (1) It focuses on connecting
(a) 1 and 2 only iconic tourist destinations.
(b) 2 and 3 only (2) It includes seaplanes for
(c) 1 and 3 only connecting Water
(d) 1, 2 and 3 Aerodromes.
(3) The Director General of Civil
Answer: (d) Aviation is the implementing
Explanation: agency for this scheme.
Statement 1 is correct: The objectives Which of the statements given
of the Deen Dayal Disabled above are correct?
Rehabilitation scheme is to create an (a) 1 and 3 only
enabling environment to ensure equal (b) 2 and 3 only
opportunities, equity, social justice and (c) 1 and 2 only
empowerment of persons with disabilities (d) 1, 2 and 3
and encourage voluntary action for
ensuring effective implementation of the Answer: (c)
People with Disabilities (Equal Explanation:
Opportunities and Protection of Statement 1 and 2 are correct: Key
Rights) Act of 1995. Features of UDAN 3.0 includes
Statement 2 is correct: The approach of 1. Inclusion of Tourism Routes under
this Scheme is to provide financial UDAN 3 in coordination with the
assistance to the voluntary organizations Ministry of Tourism
to make available the whole range of 2. Inclusion of Seaplanes for connecting
services necessary for rehabilitation of Water Aerodromes, and
persons with disabilities including early

Prelim IAS Test Series (2019) – GS Test 15 (31.03.2019)


43
Economic Survey, Budget, India Year Book, Social Schemes
3. Bringing in a number of routes in the witnesses receive appropriate and
North-East Region under the ambit of adequate protection.
UDAN (UdeDesh ka AamNagrik). Statement 2 is incorrect: The
Statement 3 is incorrect: The Airport Competent Authority under the scheme
Authority of India (AAI) is the has been defined to mean Secretary,
implementing agency of the UDAN District Legal Services Authority (DLSA)
scheme. and he/she alone can pass witness
Sources: protection order for the witness
 https://www.aai.aero/sites/default/f protection under this Scheme and who
iles/rcs_news_notifications/Press may issue orders for protection of
%20Brief%20final.pdf identity/change of identity/relocation of a
 https://www.livemint.com/Politics/ witness, categorisation of threat, duration
T9rD0myShabozUQDGWcOvO/Gu and types of protection.
wahati-likely-to-be-first-city-to-get- Statement 3 is correct: The important
international-flight.html features of the Witness Protection
Scheme, 2018 include identifying
80. With respect to the Witness categories of threat perceptions,
Protection Scheme, consider the preparation of a „Threat Analysis
following statements: Report‟ by the head of the police,
(1) It is the first attempt at the protective measures like ensuring that
national level to holistically the witness and accused do not come
provide for the protection of face to face during probe, protection of
the witnesses. identity, change of identity, relocation of
(2) The District Magistrate has witness, witnesses to be apprised of the
been defined as a competent scheme, confidentiality and preservation
authority to pass witness of records, recovery of expenses etc.
protection order. Sources:
(3) It focuses on identity  https://indianexpress.com/article/w
protection and provides for hat-is/witness-protection-scheme-
relocation with a new identity supreme-court-5480930/
to the witness.  http://www.bprd.nic.in/WriteReadD
Which of the statements given ata/CMS/Witness%20Protection%2
above are correct? 0Scheme-2018.pdf
(a) 1 and 3 only
(b) 2 and 3 only 81. Which of the following statements
(c) 1 and 2 only is/are correct regarding the „Coal
(d) 1, 2 and 3 Swapping Scheme‟?
(1) It allows State-run power
Answer: (a) generating companies to
Explanation: swap their coal supplies and
Statement 1 is correct: The Witness divert them to more efficient
Protection Scheme, 2018 (Draft) is a first plants.
attempt at the National level to holistically (2) It doesn‟t allow coal
provide for the protection of the swapping by private power
witnesses which will go a long way in producers and non-regulated
eliminating secondary victimization. This cement and steel sectors.
scheme attempts at ensuring that

Prelim IAS Test Series (2019) – GS Test 15 (31.03.2019)


44
Economic Survey, Budget, India Year Book, Social Schemes
Select the correct answer using the 82. Consider the following statements
code given below: with reference to the Pradhan
(a) 1 only Mantri Jan Dhan Yojana (PMJDY):
(b) 2 only (1) It envisages universal access
(c) Both 1 and 2 to banking facilities with at
(d) Neither 1 nor 2 least one basic banking
account for every citizen of
Answer: (a) India.
Explanation: (2) It provides an inbuilt
Statement 1 is correct: State-run power accidental insurance cover of
generating companies now have the Rupees 1 lakh on having a
flexibility to swap their coal supplies and RuPay Debit Card.
divert them to more efficient power (3) It has a provision of overdraft
plants. Coal India has signed facility up to Rs. 5000 for
agreements for aggregation of contracted every account holder.
quantity of coal with State and Central Which of the statements given
power generating companies for flexible above are incorrect?
movement of coal that would help reduce (a) 1 and 2 only
the cost of power generation. (b) 2 and 3 only
Statement 2 is incorrect: After allowing (c) 1 and 3 only
coal swapping among State-run power (d) 1, 2 and 3
plants, the Inter-Ministerial Task Force
(IMTF) has extended the supply Answer: (c)
rationalisation scheme to private power Explanation:
producers and non-regulated cement and Statement 1 is incorrect: Pradhan
steel sectors which are importing coal or Mantri Jan Dhan Yojana (PMJDY) is a
have domestic supply linkages. On National Mission on Financial Inclusion
November 26, the IMTF approved encompassing an integrated approach to
“bilateral arrangements” between two bring about comprehensive financial
willing consumers for swapping “full or inclusion of all the households in the
part of their entitled quantity” for a country. The plan envisages universal
“minimum tenure of access to banking facilities with at least
swapping/rationalisation of six months”. one basic banking account for every
Sources: household, (not every citizen) financial
 https://indianexpress.com/article/b literacy, access to credit, insurance and
usiness/economy/coal-swapping- pension facility.
scheme-extended-pvt-power- Statement 2 is correct: The
producers-cement-steel-sectors- beneficiaries would get RuPay Debit card
5505573/ having inbuilt accident insurance cover of
 https://economictimes.indiatimes.c Rs. 1 lakh. The Claim under Personal
om/industry/energy/power/coal- Accidental Insurance under PMJDY
india-allows-power-companies-to- shall be payable if the Rupay Card holder
swap- have performed minimum one successful
supplies/articleshow/58306480.cm financial or non-financial customer
s induced transaction at any Bank Branch,
Bank Mitra, ATM, POS, E-COM etc.

Prelim IAS Test Series (2019) – GS Test 15 (31.03.2019)


45
Economic Survey, Budget, India Year Book, Social Schemes
Statement 3 is incorrect: Overdraft Statement 2 is correct: The Ministry of
facility upto Rs.5000/- is available in only Health and Family Welfare, Government
one account per household, preferably of India has announced the scheme for
lady of the household. incentives for nutritional support to TB
Source: patients. The scheme is a centrally
 https://www.pmjdy.gov.in/scheme sponsored scheme under the National
 https://www.pmjdy.gov.in/about Health Mission (NHM). Financial norms
of NHM in terms of cost sharing, are
83. Regarding the „Nikshay Poshan applicable to the scheme.
Yojana‟, consider the following Statement 3 is incorrect: This scheme
statements: is implemented across all States and
(1) Under this scheme, all the TB UTs in India (not just in the States with
patients would be provided Multi Drug Resistance-Tuberculosis
the meal of two times free of (MDR-TB) population such as Uttar
cost in the hospital where Pradesh, Gujarat, Maharashtra, Madhya
they are undergoing the Pradesh, Rajasthan etc.)
treatment. Sources:
(2) It is a centrally sponsored  https://tbcindia.gov.in/showfile.ph
scheme of the Ministry of p?lid=3318
Health and Family welfare  https://www.tbfacts.org/tb-
having pan India coverage. statistics-india/
(3) The scheme is being
implemented only in the 84. Consider the following statements
states with Multi Drug with respect to the „Yuva Sahakar -
Resistance-Tuberculosis Cooperative Enterprise Support
(MDR-TB) population. and Innovation Scheme‟:
Which of the statements given (1) It has been launched to
above is/are incorrect? encourage cooperatives to
(a) 1 only venture into new and
(b) 1 and 2 only innovative areas through
(c) 1 and 3 only participation of the youth.
(d) None of the above (2) Unlike other schemes for
encouraging startups, this
Answer: (c) scheme doesn‟t give
Explanation: preferential treatment to any
Statement 1 is incorrect: It is a Direct section of the society.
Benefit Transfer (DBT) scheme for (3) All types of cooperatives,
nutritional support to Tuberculosis (TB) including those without any
patients. Under this scheme financial operational history or newly
incentive of Rs.500/-per month is established cooperatives are
provided for each notified TB patient for eligible for this scheme.
duration for which the patient is on anti- Which of the statements given
TB treatment. The incentives can be above is/are correct?
distributed in Cash (only via DBT (a) 1 only
preferably through Aadhaar enabled (b) 1 and 2 only
bank accounts) or in-kind. (c) 2 and 3 only
(d) 1, 2 and 3

Prelim IAS Test Series (2019) – GS Test 15 (31.03.2019)


46
Economic Survey, Budget, India Year Book, Social Schemes
(3) It intends to establish a
Answer: (a) three-tier network of
Explanation: pharmacovigilance centres to
Statement 1 is correct: To cater to the monitor the events
needs and aspirations of the youth, the associated with the AYUSH
National Cooperative Development drugs.
Corporation (NCDC) has come up with Select the correct answer using the
a youth-friendly scheme „Yuva Sahakar- code given below:
Cooperative Enterprise Support and (a) 2 only
Innovation Scheme‟ for attracting them (b) 1 and 2 only
to cooperative business ventures. The (c) 1 and 3 only
newly launched scheme would (d) 1, 2 and 3
encourage cooperatives to venture into
new and innovative areas. Answer: (d)
Statement 2 is incorrect: It would have Explanation:
more incentives for cooperatives of North Statement 1 is correct: The Ministry of
Eastern region, Aspirational Districts and AYUSH has introduced new Central
cooperatives with women or SC or ST or Sector Scheme for promoting
PwD members. The funding for the pharmacovigilance of Ayurveda,
project will be up to 80% of the project Siddha, Unani and Homoeopathy
cost for these special categories as (ASU&H) Drugs. The prime objective of
against 70% for others. The scheme the scheme is to develop the culture of
envisages 2% less than the applicable documenting adverse effects and
rate of interest on term loan for the undertake safety monitoring of Ayurveda,
project cost up to Rs 3 crore including 2 Siddha, Unani and Homoeopathy drugs.
years moratorium on payment of Statement 2 is correct: It will also help
principal. in surveillance of misleading
Statement 3 is incorrect: All types of advertisements appearing in the print
cooperatives in operation for at least one and electronic media. Pharmacovigilance
year are eligible for „YuvaSahakar- initiative will facilitate detection of
Cooperative Enterprise Support and potentially unsafe ASU&H medicines and
Innovation Scheme‟. misleading advertisements for taking
Source: regulatory action against them.
http://pib.nic.in/newsite/PrintRelease.a Statement 3 is correct: The scheme
spx?relid=184744 intends to facilitate the establishment of
three-tier network of National
85. Which of the following statements Pharmacovigilance Centre (NPvCC),
is/are correct regarding the Intermediary Pharmacovigilance Centres
„Scheme for pharmacovigilance of (IPvCCs) and Peripheral
AYUSH Drugs‟? Pharmacovigilance Centres (PPvCC). All
(1) It will help in developing the India Institute of Ayurveda, New Delhi, an
culture of documenting the autonomous body under the Ministry of
adverse effects of AYUSH AYUSH, has been designated as
drugs. National Pharmacovigilance Centre for
(2) It will facilitate detecting the coordinating various activities of the
misleading advertisements initiative.
regarding AYUSH drugs.

Prelim IAS Test Series (2019) – GS Test 15 (31.03.2019)


47
Economic Survey, Budget, India Year Book, Social Schemes
Source: Central/State/BADP/Local schemes
http://pib.nic.in/newsite/PrintRelease.a and participatory approach.
spx?relid=181886 Statement 3 is incorrect: BADP was
initiated in the border areas of the
86. In the context of „Border Area western region during the Seventh Five
Development Programme (BADP)‟, Year Plan period (1985-90) for ensuring
consider the following statements: balanced development of border areas
(1) It has been launched by the through development of infrastructure
Ministry of Defence to bridge and promotion of a sense of security
the gaps in socio-economic among the border population. The
infrastructure of border programme now covers 394 border
areas. blocks of 111 border districts in 17
(2) It aims to saturate the border States, which includes 167 border blocks
areas with the essential in 55 districts of 8 North East States
infrastructure through (including Sikkim), located along the
convergence of different international land border
schemes and participatory Source: https://badp.mha.gov.in/#
approach.
(3) It has been launched in the 87. Consider the following statements
light of recent escalation with respect to Impactful Policy
along India-China border Research in Social Science
after the Doklam standoff. (IMPRESS) scheme:
Which of the statements given (1) It aims at encouraging the
above is/are correct? research in ancient history of
(a) 2 only India to highlight the vast and
(b) 1 and 2 only diverse culture of India at the
(c) 1 and 3 only global level.
(d) 2 and 3 only (2) It has been launched by the
Ministry of Culture and is
Answer: (a) being implemented by the
Explanation: Indian Council of Social
Statement 1 is incorrect: The Science Research.
Department of Border Management, Which of the statements given
Ministry of Home Affairs has been above is/are correct?
implementing the Border Area (a) 1 only
Development Programme (BADP) (b) 2 only
through the State Governments as part (c) Both 1 and 2
of a comprehensive approach to Border (d) Neither 1 nor 2
Management.
Statement 2 is correct: The programme Answer: (d)
aims to meet the special development Explanation:
needs of the people living in remote and Statement 1 is incorrect: Impactful
inaccessible areas situated near the Policy Research in Social Science
international border and to saturate the (IMPRESS) aims to encourage social
border areas with the essential science research in policy relevant areas
infrastructure through convergence of so as to provide vital inputs in policy-
formulation, implementation and

Prelim IAS Test Series (2019) – GS Test 15 (31.03.2019)


48
Economic Survey, Budget, India Year Book, Social Schemes
evaluation. It has been launched to focus Statement 1 is correct: The Pradhan
research on (11) broad thematic areas Mantri Fasal Bima Yojana (PMFBY) is
such as: State and Democracy, Urban compulsory for loanee farmer obtaining
transformation, Media, Culture and Crop Loan /KCC account for notified
Society, Employment, Skills and Rural crops. However, it is voluntary for
transformation, Governance, Innovation Other/non loanee farmers who have
and Public Policy, Growth, Macro-trade insurable interest in the insured crop(s).
and Economic Policy, Agriculture and Statement 2 is correct: Pradhan Mantri
Rural Development, Health and Fasal Bima Yojana (PMFBY) aims at
Environment, Science and Education, supporting sustainable production in
Social Media and Technology, Politics, agriculture sector by way of - a) providing
Law and Economics. financial support to farmers suffering
Statement 2 is incorrect: The crop loss/damage arising out of
IMPRESS is an initiative of the Ministry unforeseen events b) stabilizing the
of Human Resource Development, income of farmers to ensure their
Government of India and is being continuance in farming c) encouraging
implemented by the Indian Council of farmers to adopt innovative and
Social Science Research. modern agricultural practices d)
Sources: ensuring flow of credit to the agriculture
 http://impress-icssr.res.in/impress/ sector; which will contribute to food
 http://vikaspedia.in/education/polic security, crop diversification and
ies-and-schemes/impress-scheme enhancing growth and competitiveness
of agriculture sector besides protecting
88. Which of the following statements farmers from production risks.
is/are correct regarding the Statement 3 is incorrect: Risks and
„Pradhan Mantri Fasal Bima Yojana Losses arising out of following perils are
(PMFBY)‟? excluded in case of PMFBY: War &
(1) A farmer has to compulsorily kindred perils, nuclear risks, riots,
subscribe to this scheme in malicious damage, theft, act of enmity,
order to obtain crop loan/ grazed and/or destroyed by domestic
Kisan Credit Card (KCC) and/or wild animals. In case of Post–
account for notified crop. Harvest losses the harvested crop
(2) It aims at encouraging bundled and heaped at a place before
farmers to adopt innovative threshing, other preventable risks.
and modern agricultural Sources:
practices.  https://pmfby.gov.in/faq
(3) It also covers losses due to  https://pmfby.gov.in/pdf/PMFBY_F
grazing by wild animals. eatures.pdf
Select the correct answer using the
code given below:
(a) 2 only 89. Consider the following pairs of
(b) 1 and 2 only schemes and their features:
(c) 1 and 3 only Scheme : Features
(d) 1, 2 and 3 1. KALIA : Cultivation
support
Answer: (b) 2. Bhavantar : Crop
Explanation: Bhugtan Insurance

Prelim IAS Test Series (2019) – GS Test 15 (31.03.2019)


49
Economic Survey, Budget, India Year Book, Social Schemes
Yojana across-country-needs-2-lakh-cr-
3. Rythu : Price say-experts/article25929784.ece
Bandhu deficiency  http://www.newindianexpress.com/
Scheme system states/odisha/2019/jan/05/what-is-
Which of the pairs given above kalia-scheme-and-who-is-eligible-
is/are correctly matched? to-get-benefit-1921003.html
(a) 1 only  https://indianexpress.com/article/i
(b) 1 and 2 only ndia/bhavantar-bhugtan-yojana-a-
(c) 2 and 3 only scheme-with-a-difference-5029247/
(d) 1 and 3 only
90. Consider the following pairs:
Answer: (a) Scheme : Objective
Explanation: (Textile
Pair 1 is correctly matched: The Sector)
(Krushak Assistance for Livelihood and 1. SAMARTH : Skill
Income Augmentation) KALIA scheme Development
will provide financial, livelihood, 2. Technology : Support for
cultivation support along with insurance Up Technical
support to small, marginal and the gradation textile
landless farmers. Financial aid of Rs. Fund Industry
25,000 per farm family over five Scheme
seasons will be provided to small and 3. PowerTex : Tax
marginal farmers so that they can incentivisation
purchase seeds, fertilizers, pesticides scheme
and use assistance towards labour. Which of the pairs given above
Pair 2 is incorrectly matched: is/are correctly matched?
Bhavantar Bhugtan Yojana is a price (a) 1 only
deficiency scheme in which the state (b) 1 and 2 only
government (e.g. Madhya Pradesh) (c) 2 and 3 only
credits the difference between the (d) 1, 2 and 3
minimum support price and the
corresponding “modal rate” (based on Answer: (b)
average prices prevailing in mandis) Explanation:
directly into the bank accounts of Pair 1 is correctly matched: SAMARTH
farmers. (Scheme For Capacity Building In Textile
Pair 3 is incorrectly matched: Under Sector) is a flagship skill development
the Rythu Bandhu scheme being scheme approved by the Cabinet
implemented in Telangana, every Committee of Economic Affairs (CCEA)
landholding farmer is given a direct in continuation to the Integrated Skill
benefit of ₹4,000 per acre per crop Development Scheme for 12th Five Year
season as investment support Plan.
irrespective of his/her extent of Pair 2 is correctly matched: All
landholding. technical textile machinery is covered
Sources: under the Technology Upgradation
 https://www.thehindu.com/news/ci Fund Scheme (TUFS). Under TUFS,
ties/Hyderabad/rythu-bandhu- specified technical textile machinery has
been provided with additional benefit in

Prelim IAS Test Series (2019) – GS Test 15 (31.03.2019)


50
Economic Survey, Budget, India Year Book, Social Schemes
terms of 10% capital subsidy in addition science literacy every three
to 5% interest reimbursement years.
Pair 3 is incorrectly matched: The (d) An international assessment
Government has launched PowerTex that measures students‟
India, a comprehensive scheme for fitness and extra-curricular
powerloom sector development. It participation.
includes:
 In-situ Upgradation of Plain Answer: (c)
Powerlooms Explanation:
 Group Workshed Scheme (GWS) The Human Resource Development
 Yarn Bank Scheme (HRD) ministry has signed an agreement
 Common Facility Centre (CFC) with the Organisation for Economic
 Pradhan Mantri Credit Scheme for Co-operation and Development
Powerloom Weavers (OECD) which conducts PISA every
 Solar Energy Scheme for three years confirming India‟s
Powerlooms participation in the triennial global survey
 Facilitation, IT, Awareness, Market that assesses 15-year old students in
Development and Publicity for reading, mathematics, science and
Powerloom Schemes collaborative problem-solving.
Participation in PISA allows
 Tex Venture Capital Fund
benchmarking performance against a
 Grant-in-Aid and Modernisation &
wide range of countries. PISA uses test
Upgradation of Powerloom Service
items aligned with international
Centres (PSCs)
benchmarks. Test items are adapted to
Source:
the local context and language, pilot
 https://samarth-textiles.gov.in/ tested and validated.
 http://www.texmin.nic.in/sites/defa PISA is a competency-based
ult/files/scheme_technical_textile_ assessment which, unlike content-based
070116.pdf assessment, measures the extent to
 http://pib.nic.in/newsite/PrintRelea which students have acquired key
se.aspx?relid=160404 competencies that are essential for full
participation in modern societies.
91. The Programme for International Hence, option (c) is the correct
Student Assessment (PISA) answer.
coordinated by the Organization for Source:
Economic Cooperation and https://indianexpress.com/article/educ
Development (OECD) is ation/pisa-tests-india-to-take-part-in-
(a) An international assessment global-teen-learning-test-in-2021-
of collegiate education of 4537231/
major universities worldwide.
(b) An international benchmark 92. With reference to the „Pradhan
for ranking the education Mantri Shram Yogi Maan-Dhan
system of participating Yojana (PM-SYM)‟, consider the
nations. following statements:
(c) An international assessment (1) PM-SYM is a voluntary and
that measures students contributory pension scheme.
reading, mathematics, and

Prelim IAS Test Series (2019) – GS Test 15 (31.03.2019)


51
Economic Survey, Budget, India Year Book, Social Schemes
(2) It includes both organised Hence, option (c) is the correct
and unorganised sector answer.
workers as beneficiaries. Sources:https://www.ndtv.com/busine
(3) Workers of all age groups are ss/pradhan-mantri-shram-yogi-maan-
eligible for this scheme. dhan-yojna-pm-sym-scheme-monthly-
(4) It is implemented through Life pension-contributions-benefits-
Insurance Corporation of 2003448
India and CSC eGovernance
Services India Limited (CSC 93. With reference to the „Pradhan
SPV). Mantri Kisan Samman Nidhi (PM-
Which of the statements given KISAN)‟, consider the following
above are incorrect? statements?
(a) 1 and 2 only (1) PM-KISAN will benefit both
(b) 1 and 4 only small land holders and the
(c) 2 and 3 only tenant farmers.
(d) 2, 3 and 4 only (2) Beneficiaries would be
identified as per the central
Answer: (c) land record databases.
Explanation: (3) Direct monetary supports like
PM-SYM is a voluntary and contributory this can invite objections at
pension scheme that will engage as WTO, as it is an amber box
many as 42 crore workers in the subsidy.
unorganised sector. (4) Aadhaar has now been made
Eligibility for PM-SYM: mandatory to avail any
1. The unorganised sector workers, with benefit under the scheme.
income of less than Rs 15,000 per Which of the statements given
month and who belong to the entry above is/are incorrect?
age group of 18-40 years, will be (a) 1 only
eligible for the scheme. (b) 1 and 2 only
2. Those workers should not be covered (c) 2, 3 and 4 only
under New Pension Scheme (NPS), (d) 1, 2, 3 and 4
Employees‟ State Insurance
Corporation (ESIC) scheme or Answer: (d)
Employees‟ Provident Fund Explanation:
Organisation (EPFO). The PM-KISAN scheme aims to
3. He or she should not be an income transform the lives of small and marginal
tax payer. farmers in India, by providing them
PM-SYM is a Central government assured annual monetary support of Rs.
scheme administered by the Ministry of 6000 in three equal instalments. The
Labour and Employment and scheme envisages providing income
implemented through Life Insurance support to supplement the financial
Corporation of India and CSC needs of the eligible farmers in procuring
eGovernance Services India Limited various inputs to ensure proper crop
(CSC SPV). LIC will be the Pension Fund health and appropriate yields,
Manager and responsible for Pension commensurate with the anticipated farm
pay out. yields.
Features:

Prelim IAS Test Series (2019) – GS Test 15 (31.03.2019)


52
Economic Survey, Budget, India Year Book, Social Schemes
1. All the farmers who have land holdings (c) Charging infrastructure has
upto 2 hectare are eligible to get the been kept out of this
benefit of the scheme. Direct Benefit scheme‟s purview.
Transfer at Rs. 6000 per annum per (d) It will contribute towards Paris
eligible family shall be made in three climate deal commitments.
equal instalments of Rs. 2000 each.
Landless farmers, tenant farmers and Answer: (c)
sharecroppers have been left out of it‟s Explanation:
purview. Hence, statement 1 is The Faster Adoption and Manufacturing
incorrect. of (Hybrid &) Electric Vehicles (FAME)
2. For identification of beneficiary, the India was launched in 2015 under
Centre has defined a small and marginal National Electric Mobility Mission
landholder family as the one comprising (NEMM). It aims at promoting eco-
of husband, wife and minor children up to friendly vehicles in the country.
18 years of age, who collectively own Option (a) is correct. It envisages
cultivable land up to two hectares as per support to hybrid or electric vehicles
the land records of the concerned states. market development and its
The number of eligible SMFs under the manufacturing eco-system in country in
scheme has been estimated on the basis order to achieve self-sustenance in
of projection of Agricultural Census 2015- stipulated period. It covers hybrid and
16 data for 2018-19. Hence, statement electric technologies like a strong
2 is incorrect. hybrid, plug-in hybrid and battery
3. It is income support model, which electric vehicles.
does not distort prices, and comes Option (b) is correct. The scheme is
under blue-box subsidy. Hence, being administered by the Heavy
statement 3 is incorrect. Industries Ministry.
4. Aadhaar is optional for getting first Option (c) is incorrect. 4 focus areas of
instalment, but has been made scheme (within its purview) are -
mandatory for second instalment Technology development, demand
onwards. Hence, statement 4 is creation, pilot projects, and charging
incorrect. infrastructure.
Source:https://www.pmkisan.nic.in/Do Option (d) is correct: Electric mobility
cuments/PMKisanSamanNidhi.PDF will help reduce carbon dioxide
emissions from all types of automotive
94. Regarding the Faster Adoption and vehicles, which will contribute towards
Manufacturing of (Hybrid &) INDC and Paris climate deal.
Electric Vehicles in India (FAME- Hence, option (c) is the correct
India) scheme, which of the answer.
following statements is incorrect? Source: https://fame-india.gov.in/.
(a) It envisages support to
hybrid/electric vehicles 95. In the context of „Ethanol Blending
market development and Programme (EBP)‟ in India,
Manufacturing ecosystem in consider the following statements:
India. Assertion: Ethanol blended
(b) The scheme is being gasoline has a higher-octane rating
administered by the Heavy than the normal gasoline.
Industries Ministry.

Prelim IAS Test Series (2019) – GS Test 15 (31.03.2019)


53
Economic Survey, Budget, India Year Book, Social Schemes
Reason: The compression ratio of of Tourism, whereas HRIDAY
the engine using such a fuel could is being implemented by the
be safely increased for increased Ministry of Culture.
thermal efficiency. (2) PRASAD scheme has a
Select the correct answer using the cultural dimension whereas
code given below: HRIDAY scheme has a
(a) Both Assertion and Reason religious dimension.
are true and Reason is the Which of the statements given
correct explanation of above is/are correct?
Assertion. (a) 1 only
(b) Both Assertion and Reason (b) 2 only
are true but Reason is not the (c) Both 1 and 2
correct explanation of (d) Neither 1 nor 2
Assertion.
(c) Assertion is true but Reason Answer: (d)
is false. Explanation:
(d) Assertion is false but Reason Statement 1 is incorrect. PRASAD is
is true. being implemented by the Ministry of
Tourism, whereas HRIDAY is being
Answer: (b) implemented by Ministry of Housing &
Explanation: Urban Affairs.
Ethanol does not easily vaporize, hence Statement 2 is incorrect. PRASAD
it can be compressed to higher ratios, aims at integrated development of
thus providing for higher thermal pilgrimage destinations in planned,
efficiency of automotive engines. The prioritised and sustainable manner to
mixture is less prone to premature provide complete religious tourism
ignition when compressed, hence it‟s experience. It focuses on the
octane rating is higher than pure development and beautification of the
gasoline. identified pilgrimage destinations. The
Hence, both assertion and reason are National Heritage Development and
correct, but reason is not the correct Augmentation Yojana (HRIDAY) seeks to
explanation of assertion. In fact, preserve and rejuvenate the rich cultural
Assertion correctly explains the Reason. heritage of the country. HRIDAY seeks to
So, Option (b) is the correct answer. promote an integrated, inclusive and
Source: sustainable development of heritage
http://www.arthapedia.in/index.php?tit sites, focusing not just on maintenance of
le=Ethanol_Blending_Programme_ monuments but on advancement of the
(EBP)_in_India entire ecosystem including its citizens,
tourists and local businesses. Thus,
96. Consider the following features both PRASAD and HRIDAY schemes
about Pilgrimage Rejuvenation and have cultural and religious
Spiritual Augmentation Drive dimensions.
(PRASAD) and Heritage City Source: India Year Book 2018, Chapter
Development and Augmentation 5 - Culture and Tourism, Page 72.
Yojana (HRIDAY) schemes:
(1) PRASAD is being 97. Which of the following statements
implemented by the Ministry is/are correct with reference to the

Prelim IAS Test Series (2019) – GS Test 15 (31.03.2019)


54
Economic Survey, Budget, India Year Book, Social Schemes
„Scheme for Higher Education employable skills into their learning,
Youth in Apprenticeship and Skills promote apprenticeship as integral to
(SHREYAS)‟? education and also amalgamate
(1) It is being implemented by employment facilitating efforts of the
the Ministry of Micro Small Government into the education system
and Medium Enterprises so that clear pathways towards
(MSME). employment opportunities are available
(2) The programme promotes to students during and after their
apprenticeship as an graduation. Following are the
important part of objectives of SHREYAS:
education and integrates the  To improve employability of students
government‟s employment by introducing employment relevance
building efforts into the into the learning process of the
education. higher education system
(3) It will focus on students  To forge a close functional link
primarily from non-technical between education and
courses. industry/service sectors on a
(4) Educational institutions and sustainable basis
industry could provide their  To provide skills which are in
respective demand and demand, to the students in a dynamic
supply of apprenticeship on manner
it‟s platform.  To establish an 'earn while you learn'
Select the correct answer using the system into higher education
code given below:  To help business/industry in securing
(a) 1 only good quality manpower
(b) 1 and 2 only  To link student community with
(c) 2, 3 and 4 only employment facilitating efforts of the
(d) 1, 2, 3 and 4 Government
Hence, option (c) is the correct
Answer: (c) answer.
Explanation: Sources:
Statement 1 is incorrect: SHREYAS is https://shreyas.ac.in/Home/about
a programme basket comprising the
initiatives of three Central Ministries, 98. Regarding the Atal Innovation
namely the Ministry of Human Resource Mission (AIM), which of the
Development, Ministry of Skill following statements are correct?
Development & Entrepreneurship and (1) Atal Innovation Mission
the Ministry of Labour& Employment viz. (AIM) along with Self-
the National Apprenticeship Promotion Employment and Talent
Scheme (NAPS), the National Career Utilization (SETU) is a
Service (NCS) and introduction of Government of India‟s
BA/BSc/BCom (Professional) courses in endeavour to promote a
the higher educational institutions. culture of innovation and
Statements 2, 3 and 4 are correct: entrepreneurship.
SHREYAS is a programme conceived for (2) It has been setup under the
students in degree courses, primarily Department of Science and
non-technical, with a view to introduce Technology.

Prelim IAS Test Series (2019) – GS Test 15 (31.03.2019)


55
Economic Survey, Budget, India Year Book, Social Schemes
(3) It is an umbrella scheme computational thinking, adaptive
wherein the innovators would learning and artificial intelligence.
be supported and mentored Source:
as successful entrepreneurs. https://aim.gov.in/overview.php
(4) Atal Tinkering Labs (ATL)
have been envisaged to be 99. Which of the following is not
setup in schools and colleges correct regarding the National
across the nation. Rural Economic Transformation
Select the correct answer using the Project (NRETP)?
code given below: (a) It will be funded with support
(a) 1 and 4 only from the World Bank.
(b) 1 and 3 only (b) It focuses on women-led farm
(c) 2 and 3 only and non-farm initiatives.
(d) 1, 2 and 4 only (c) It was originally called as
Ajeevika.
Answer: (b) (d) The project aims to provide
Explanation: interventions to enhance the
Statements 1 and 3 are correct. With a livelihoods promotion access
view to give substantial boost to the to finance and scale-up
innovation ecosystem and to catalyse the initiatives on digital finance
entrepreneurial spirit in the country, the and livelihood interventions.
Atal Innovation Mission (AIM) and Self
Employment and Talent Utilisation Answer: (c)
(SETU) have been setup in NITI Aayog Explanation:
with appropriate manpower. AIM is a The Union Government has proposed a
Mission and SETU is an approach. new rural Livelihood Scheme, the
Therefore, there is one umbrella scheme National Rural Economic Transformation
titled as 'AIM' which will have two sub- Project under the Deendayal Antyodaya
components- (i) Innovation; and (ii) Yojana - National Rural Livelihoods
SETU, wherein the innovators would be Mission (DAY-NRLM).
supported and mentored as successful National Rural Economic
entrepreneurs. Transformation Project -
Statements 2 and 4 are incorrect. Atal  Option (a) is correct. The project
Tinkering Labs are dedicated works would be undertaken with loan
spaces where students (Class 6th to assistance from the World Bank.
Class 12th) learn innovation skills and Government of India and World Bank
develop ideas that will go on to transform have Signed a $250 Million
India. The labs are powered to acquaint Agreement for the National Rural
students with state-of-the-art Economic Transformation Project
equipment such as 3D printers, (NRETP) to boost Rural Incomes
robotics & electronics development across 13 States in India.
tools, IoT & sensors etc. The lab  Option (b) is correct. A Key Focus
activities are designed to spur the spark of the Project will be to promote
of creativity, and go beyond regular women-owned and women-led farm
curriculum and text book learning. The and non-farm enterprises across
labs will let students explore skills of value chains; enable them to build
future such as design and businesses that help them access

Prelim IAS Test Series (2019) – GS Test 15 (31.03.2019)


56
Economic Survey, Budget, India Year Book, Social Schemes
finance, markets and networks; and  Targeted Beneficiaries: All girl
generate employment. children within the age of 13 to 19
 Option (c) is incorrect. Ajeevika is years in the state.
the older name for Deendayal  The scheme was awarded United
Upadhyaya Antyodaya Yojana - Nations Public Service Award in June
National Rural Livelihood Mission 2017.
(DAY-NRLM) and not NRETP.  Hence, the correct answer is
 Option (d) is correct: The project option (b).
aims to provide interventions to  Source:
enhance the livelihoods promotion https://wbkanyashree.gov.in/kp_sc
access to finance and scale-up heme.php
initiatives on digital finance and
livelihood interventions.
Source:
http://pib.nic.in/newsite/PrintRelease.a
spx?relid=189206

100. Kanyashree Prakalpa Yojana aims


at incentivizing schooling of all
teenage girls and delaying their
marriages until the age of 18 years,
the legal age of marriage. Which of
the following Indian State has
launched this scheme?
(a) Odisha
(b) West Bengal
(c) Bihar
(d) Chhattisgarh

Answer: (b)
Explanation:
Kanyashree Prakalpa scheme - It is a
conditional cash transfer scheme
aimed at improving status and well-being
of girl child by incentivising schooling
ofall teenage girls and delaying their
marriages until the age of 18 years, the
legal age of marriage.
 It was launched by Mamata Banerjee
led West Bengal Government
(Women Development and Social
Welfare department) in October
2013.
 It was aimed at increasing
educational attainment of girls,
prevention of child marriage and
financial inclusion.

Prelim IAS Test Series (2019) – GS Test 15 (31.03.2019)


57
Economic Survey, Budget, India Year Book, Social Schemes

Das könnte Ihnen auch gefallen